You are on page 1of 52

Territory

(A. The Philippine Territory and the


archipelagic doctrine
B. Law of the Seas
C. National Economy and Patrimony)

1
1. Tañada vs. Angara, 272 SCRA 18
(Petitioners were members of Congress and as taxpayers and NGOs)

DOCTRINE:
Where an action of the legislative branch is seriously alleged to have infringed the Constitution, it becomes
not only the right but in fact the duty of the judiciary to settle the dispute. The duty to adjudicate
remains to assure that the supremacy of the Constitution is upheld. Once a controversy as to the
application or interpretation of a constitutional provision is raised before this Court, it becomes a
legal issue which the Court is bound by constitutional mandate to decide.

FACTS:
This is a petition seeking to nullify the Philippine ratification of the World Trade Organization (WTO)
Agreement. Petitioners question the concurrence of herein respondents acting in their capacities as Senators
via signing the said agreement.

On April 15, 1994, the Philippine Government represented by its Secretary of the Department of Trade and
Industry, Rizalino Navarro, signed the Final Act binding the Philippine Government to submit to its
respective competent authorities the WTO (World Trade Organization) Agreements to seek approval for
such. On December 14, 1994, Resolution No. 97 was adopted by the Philippine Senate to ratify the WTO
Agreement.

The WTO opens access to foreign markets, especially its major trading partners, through the reduction of
tariffs on its exports, particularly agricultural and industrial products. Thus, provides new opportunities for
the service sector cost and uncertainty associated with exporting and more investment in the country. These
are the predicted benefits as reflected in the agreement and as viewed by the signatory Senators, a "free
market" espoused by WTO.

Petitioners on the other hand viewed the WTO agreement as one that limits, restricts and impair Philippine
economic sovereignty and legislative power. Petitioners assail the constitutionality of the WTO agreement
as it violates Sec 19, Article II, providing for the development of a self-reliant and independent national
economy, and Sections 10 and 12, Article XII, providing for the "Filipino first" policy. That the Filipino
First policy of the Constitution was taken for granted as it gives foreign trading intervention.

ISSUE/S:
(1) WHETHER OR NOT THE PETITION INVOLVES A POLITICAL QUESTION.
(2) WHETHER OR NOT THE WTO AGREEMENT CONTRAVENE SEC. 19, ARTICLE II, AND
SECS. 10 AND 12, ARTICLE XII, OF THE PHILIPPINE CONSTITUTION.

HELD:
(1) NO. The petition raises a justiciable controversy. Where an action of the legislative branch is seriously
alleged to have infringed the Constitution, it becomes not only the right but in fact the duty of the judiciary
to settle the dispute. The duty to adjudicate remains to assure that the supremacy of the Constitution is
upheld. Once a controversy as to the application or interpretation of a constitutional provision is raised
before this Court, it becomes a legal issue which the Court is bound by constitutional mandate to decide.
In deciding to take jurisdiction over this petition, this Court will not review the wisdom of the decision of
the President and the Senate in enlisting the country into the WTO, or pass upon the merits of trade
liberalization as a policy espoused by said international body. Neither will it rule on the propriety of the
government's economic policy of reducing/removing tariffs, taxes, subsidies, quantitative restrictions, and
other import/trade barriers. Rather, it will only exercise its constitutional duty "to determine whether or
not there had been a grave abuse of discretion amounting to lack or excess of jurisdiction" on the
part of the Senate in ratifying the WTO Agreement and its three annexes.

(2) NO. Article II of the Constitution, "declaration of principles and state policies", are not intended to be

2
self-executing principles ready for enforcement through the courts. They are used by the judiciary as aids or
as guides in the exercise of its power of judicial review, and by the legislature in its enactment of laws.
They do not embody judicially enforceable constitutional rights but guidelines for legislation." Broad
constitutional principles need legislative enactments to implement them. They were rather directives
addressed to the executive and to the legislature. If the executive and the legislature failed to heed the
directives of the article, the available remedy was not judicial but political.

On the other hand, Secs. 10 and 12 of Article XII, apart from merely laying down general principles relating
to the national economy and patrimony, should be read and understood in relation to the other sections in
said article, especially Secs. 1 and 13 thereof. With these goals in context, the Constitution then ordains
the ideals of economic nationalism (1) by expressing preference in favor of qualified Filipinos "in the
grant of rights, privileges and concessions covering the national economy and patrimony" and in the use
of "Filipino labor, domestic materials and locally-produced goods"; (2) by mandating the State to "adopt
measures that help make them competitive; and (3) by requiring the State to "develop a self-reliant and
independent national economy effectively controlled by Filipinos."

It is true that in the recent case of Manila Prince Hotel vs. Government Service Insurance System, et al, this
Court held that "Sec. 10, second par., Art. XII of the 1987 Constitution is a mandatory, positive command
which is complete in itself and which needs no further guidelines or implementing laws or rules for its
enforcement. From its very words the provision does not require any legislation to put it in operation. It is
per se judicially enforceable." However, as the constitutional provision itself states, it is enforceable only in
regard to "the grants of rights, privileges and concessions covering national economy and patrimony"
and not to every aspect of trade and commerce. It refers to exceptions rather than the rule. The issue
here whether, as a rule, there are enough balancing provisions in the Constitution to allow the Senate to
ratify the Philippine concurrence in the WTO Agreement. And we hold that there are.

The Constitution did not intend to pursue an isolationist policy. It did not shut out foreign investments,
goods and services in the development of the Philippine economy. While the Constitution does not
encourage the unlimited entry of foreign goods, services and investments into the country, it does not
prohibit them either. In fact, it allows an exchange on the basis of equality and reciprocity, frowning only
on foreign competition that is unfair.

3
2. Garcia vs. Corona, G.R. 132451, December 17, 1999

Facts:

After RA 8180 (An Act Deregulating the Downstream Oil Industry and For Other Purposes)
was declared unconstitutional in Tatad vs Sec. of DOE, Congress enacted Republic Act No. 8479, a
new deregulation law without the offending provisions of the earlier law. Petitioner Enrique T.
Garcia, a member of Congress, has now brought this petition seeking to declare Section 19 thereof,
which sets the time of full deregulation, unconstitutional. After failing in his attempts to have
Congress incorporate in the law the economic theory he espouses, petitioner now asks us, in the
name of upholding the Constitution, to undo a violation which he claims Congress has committed.
Petitioner contends that Section 19 of R.A. 8479, which prescribes the period for the removal
of price control on gasoline and other finished products and for the full deregulation of the local
downstream oil industry, is patently contrary to public interest and therefore unconstitutional
because within the short span of five months, the market is still dominated and controlled by an
oligopoly of the three (3) private respondents, namely, Shell, Caltex and Petron.
The objective of the petition is deceptively simple. It states that if the constitutional mandate
against monopolies and combinations in restraint of trade[2] is to be obeyed, there should be
indefinite and open-ended price controls on gasoline and other oil products for as long as
necessary. This will allegedly prevent the Big 3 --- Shell, Caltex and Petron --- from price-fixing
and overpricing. Petitioner calls the indefinite retention of price controls as partial deregulation.
Be that as it may, we are not concerned with whether or not there should be deregulation. This
is outside our jurisdiction. The judgment on the issue is a settled matter and only Congress can
reverse it. Rather, the question that we should address here is --- are the method and the manner
chosen by Government to accomplish its cherished goal offensive to the Constitution? Is indefinite
price control in the manner proposed by petitioner the only feasible and legal way to achieve it?
In his recital of the antecedent circumstances, petitioner repeats in abbreviated form the factual
findings and conclusions which led the Court to declare R.A. 8180 unconstitutional. The foreign
oligopoly or cartel formed by respondents Shell, Caltex and Petron, their indulging in price-fixing
and overpricing, their blockade tactics which effectively obstructed the entry of genuine
competitors, the dangers posed by the oil cartel to national security and economic development, and
other prevailing sentiments are stated as axiomatic truths. They are repeated in capsulized context
as the current background facts of the present petition.
The Court respects the legislative finding that deregulation is the policy answer to the
problems. It bears stressing that R.A. 8180 was declared invalid not because deregulation is
unconstitutional. The law was struck down because, as crafted, three key provisions plainly
encouraged the continued existence if not the proliferation of the constitutionally proscribed evils of
monopoly and restraint of trade.
In sharp contrast, the present petition lacks a factual foundation specifically highlighting the
need to declare the challenged provision unconstitutional. There is a dearth of relevant, reliable, and
substantial evidence to support petitioners theory that price control must continue even as
Government is trying its best to get out of regulating the oil industry. The facts of the petition are, in
the main, a general dissertation on the evils of monopoly.
A dramatic, at times expansive and grandiloquent, reiteration of the same background
circumstances narrated in Tatad does not squarely sustain petitioners novel thesis that there can be
deregulation without lifting price controls.
Petitioner may call the industry subject to price controls as deregulated. In enacting the
challenged provision, Congress, on the other hand, has declared that any industry whose prices and
profits are fixed by government authority remains a highly regulated one.
Petitioner, therefore, engages in a legal paradox. He fails to show how there can be
deregulation while retaining government price control. Deregulation means the lifting of control,
governance and direction through rule or regulation. It means that the regulated industry is freed

4
from the controls, guidance, and restrictions to which it used to be subjected. The use of the word
partial to qualify deregulation is sugar-coating. Petitioner is really against deregulation at this time.
Petitioner engages in another contradiction when he puts forward what he calls a self-evident
truth. He states that a truly competitive market and fair prices cannot be legislated into
existence. However, the truly competitive market is not being created or fashioned by the
challenged legislation. The market is simply freed from legislative controls and allowed to grow
and develop free from government interference. R.A. 8479 actually allows the free play of supply
and demand to dictate prices. Petitioner wants a government official or board to continue
performing this task. Indefinite and open-ended price control as advocated by petitioner would be to
continue a regime of legislated regulation where free competition cannot possibly flourish. Control
is the antithesis of competition. To grant the petition would mean that the Government is not keen
on allowing a free market to develop. Petitioners self-evident truth thus supports the validity of the
provision of law he opposes.

We are not impressed by petitioner Garcias submission. Petitioner has no basis in condemning as
unconstitutional per se the date fixed by Congress for the beginning of the full deregulation of the
downstream oil industry. Our Decision merely faulted the Executive for factoring the depletion of
OPSF in advancing the date of full deregulation to February 1997. Nonetheless, the error of the
Executive is now a non-issue for the full deregulation set by Congress itself at the end of March
1997 has already come to pass. March 1997 is not an arbitrary date. By that date, the transition
period has ended and it was expected that the people would have adjusted to the role of market
forces in shaping the prices of petroleum and its products. The choice of March 1997 as the date of
full deregulation is a judgment of Congress and its judgment call cannot be impugned by this
Court.[8]

Reduced to its basic arguments, it can be seen that the challenge in this petition is not against
the legality of deregulation. Petitioner does not expressly challenge deregulation.The issue, quite
simply, is the timeliness or the wisdom of the date when full deregulation should be effective.
In this regard, what constitutes reasonable time is not for judicial determination. Reasonable
time involves the appraisal of a great variety of relevant conditions, political, social and
economic. They are not within the appropriate range of evidence in a court of justice. It would be
an extravagant extension of judicial authority to assert judicial notice as the basis for the
determination.[9]
We repeat that what petitioner decries as unsuccessful is not a final result. It is only a
beginning. The Court is not inclined to stifle deregulation as enacted by Congress from its very
start. We leave alone the program of deregulation at this stage. Reasonable time will prove the
wisdom or folly of the deregulation program for which Congress and not the Court is
accountable.
It can be seen, therefore, that instead of the price controls advocated by the petitioner,
Congress has enacted anti-trust measures which it believes will promote free and fair
competition. Upon the other hand, the disciplined, determined, consistent and faithful execution of
the law is the function of the President. As stated by public respondents, the remedy against
unreasonable price increases is not the nullification of Section 19 of R.A. 8479 but the setting into
motion of its various other provisions.
For this Court to declare unconstitutional the key provision around which the laws anti-trust
measures are clustered would mean a constitutionally interdicted distrust of the wisdom of Congress
and of the determined exercise of executive power.
Having decided that deregulation is the policy to follow, Congress and the President have
the duty to set up the proper and effective machinery to ensure that it works. This is something
which cannot be adjudicated into existence. This Court is only an umpire of last resort whenever the
Constitution or a law appears to have been violated. There is no showing of a constitutional
violation in this case.
WHEREFORE, the petition is DISMISSED. SO ORDERED.
5
3. Province of North Cotabato vs. GRP Peace Panel on Ancestral Domain, et. al., G.R. No. 183591,
October 14, 2008

FACTS:
On August 5, 2008, the Government of the Republic of the Philippines (GRP) and the MILF, through the
Chairpersons of their respective peace negotiating panels, were scheduled to sign a Memorandum of
Agreement on the Ancestral Domain (MOA-AD) Aspect of the GRP-MILF Tripoli Agreement on Peace of
2001 in Kuala Lumpur, Malaysia.
The signing of the MOA-AD between the GRP and the MILF was not to materialize, however, for upon
motion of petitioners, specifically those who filed their cases before the scheduled signing of the MOA- AD,
the SC issued a Temporary Restraining Order enjoining the GRP from signing the same. Formal peace talks
between the parties were held in Tripoli, Libya from June 20-22, 2001, the outcome of which was the GRP-
MILF Tripoli Agreement on Peace (Tripoli Agreement 2001) containing the basic principles and agenda on
the following aspects of the negotiation: Security Aspect, Rehabilitation Aspect, and Ancestral Domain
Aspect. With regard to the Ancestral Domain Aspect, the parties in Tripoli Agreement 2001 simply agreed
"that the same be discussed further by the Parties in their next meeting."
On July 23, 2008, the Province of North Cotabato and Vice-Governor Emmanuel Pinol filed a petition,
docketed as G.R. No. 183591, for Mandamus and Prohibition with Prayer for the Issuance of Writ of
Preliminary Injunction and Temporary Restraining Order.
This initial petition was followed by another one, docketed as G.R. No. 183752, also for Mandamus and
Prohibition filed by the City of Zamboanga, Mayor Celso Lobregat, Rep. Ma. Isabelle Climaco and Rep.
Erico Basilio Fabian who likewise pray for similar injunctive reliefs.
By Resolution of August 4, 2008, the Court issued a Temporary Restraining Order commanding and
directing public respondents and their agents to cease and desist from formally signing the MOA-AD.
Meanwhile, the City of Iligan filed a petition for Injunction and/or Declaratory Relief, docketed as G.R. No.
183893, praying that respondents be enjoined from signing the MOA-AD or, if the same had already been
signed, from implementing the same, and that the MOA-AD be declared unconstitutional.
The Province of Zamboanga del Norte, Governor Rolando Yebes, Vice-Governor Francis Olvis, Rep. Cecilia
Jalosjos-Carreon, Rep. Cesar Jalosjos, and the members18 of the Sangguniang Panlalawigan of Zamboanga
del Norte filed on August 15, 2008 a petition for Certiorari, Mandamus and Prohibition,19 docketed as G.R.
No. 183951
By subsequent Resolutions, the Court ordered the consolidation of the petitions.

ISSUES:
WON the MOA-AD grants to the Bangsamoro Juridical Entity (BJE) the status of a STATE
WON the BJE contains the 4 elements of a State

HELD:
The MOA-AD proceeds to refer to the "Bangsamoro homeland," the ownership of which is vested
exclusively in the Bangsamoro people by virtue of their prior rights of occupation. Both parties to the MOA-
AD acknowledge that ancestral domain does not form part of the public domain.
The territory of the Bangsamoro homeland is described as the land mass as well as the maritime, terrestrial,
fluvial and alluvial domains, including the aerial domain and the atmospheric space above it, embracing the
Mindanao- Sulu-Palawan geographic region.
More specifically, the core of the BJE is defined as the present geographic area of the ARMM - thus
constituting the following areas: Lanao del Sur, Maguindanao, Sulu, Tawi-Tawi, Basilan, and Marawi City.
Significantly, this core also includes certain municipalities of Lanao del Norte that voted for inclusion in the
ARMM in the 2001 plebiscite.
These provisions of the MOA indicate, among other things, that the Parties aimed to vest in the BJE the
status of an associated state or, at any rate, a status closely approximating it. The concept of association is
not recognized under the present Constitution No province, city, or municipality, not even the ARMM, is
recognized under our laws as having an "associative" relationship with the national government. Indeed, the
concept implies powers that go beyond anything ever granted by the Constitution to any local or regional
government. It also implies the recognition of the associated entity as a state. The Constitution, however,
6
does not contemplate any state in this jurisdiction other than the Philippine State, much less does it provide
for a transitory status that aims to prepare any part of Philippine territory for independence.

The BJE is a far more powerful entity than the autonomous region recognized in the Constitution It is
not merely an expanded version of the ARMM, the status of its relationship with the national government
being fundamentally different from that of the ARMM. Indeed, BJE is a state in all but name as it meets the
criteria of a state laid down in the Montevideo Convention, namely, a permanent population, a defined
territory, a government, and a capacity to enter into relations with other states.
Even assuming arguendo that the MOA-AD would not necessarily sever any portion of Philippine territory,
the spirit animating it - which has betrayed itself by its use of the concept of association - runs counter to the
national sovereignty and territorial integrity of the Republic.
The defining concept underlying the relationship between the national government and the BJE being itself
contrary to the present Constitution, it is not surprising that many of the specific provisions of the MOA- AD
on the formation and powers of the BJE are in conflict with the Constitution and the laws.

7
4. RP v China

FACTS:

The Republic of the Philippines(Philippines) instituted an arbitration case against the People’s Republic of
China(China) under the 1982 United Nations Convention on the Law of the Sea(Convention or UNCLOS)
since both parties have ratified the Convention. However, China have consistently stated its view on the
lack of jurisdiction of the Tribunal on the matter.
The arbitration concerns disputed between the parties regarding the legal basis of maritime rights and
entitlements in the South China Sea, the status of certain geographic features in the South China Sea, and
the lawfulness of certain actions taken by China in the South China Sea.

ISSUES:
1. WON the Tribunal has jurisdiction.
2. Whether the case is not a sovereignty dispute
3. Whether China have claims under historical rights and the “nine-dash-line”
4. What is the status of features in the South China Sea
4. WON the activities of China in the South China Sea is lawful.
5. WON the actions of China since the commencement of arbitration have aggravated and extended the
dispute.
6. What is China’s future conduct?

RULING:

1. Article 288 of the Conventions states that “In the event of a dispute as to whether a court or tribunal has
jurisdiction, the matter shall be settled by decision of that court or tribunal.”
PRC states that as PRC did not participate in the case in any stage, the tribunal has no jurisdiction over it.
Thus, the case must be dismissed.

2.

8
3. With respect to Submission No. 1, for the reasons set out above, the Tribunal concludes that, as between
the Philippines and China, the Convention defines the scope of maritime entitlements in the South China
Sea, which may not extend beyond the limits imposed therein.
the Tribunal concludes that, as between the Philippines and China, China’s claims to historic rights, or
other sovereign rights or jurisdiction, with respect to the maritime areas of the South China Sea
encompassed by the relevant part of the ‘nine-dash line’ are contrary to the Convention and without lawful
effect to the extent that they exceed the geographic and substantive limits of China’s maritime entitlements
under the Convention. The Tribunal concludes that the Convention superseded any historic rights or other
sovereign rights or jurisdiction in excess of the limits imposed therein.

4.

5. the Tribunal finds that China has, by virtue of the conduct of Chinese law enforcement vessels in the
vicinity of Scarborough Shoal, created serious risk of collision and danger to Philippine vessels and
personnel. The Tribunal finds China to have violated Rules 2, 6, 7, 8, 15, and 16 of the COLREGS and, as
a consequence, to be in breach of Article 94 of the Convention.

6. yes, it has.

(a) China has aggravated the Parties’ dispute concerning their respective rights and entitlements in the area
of Mischief Reef by building a large artificial island on a low-tide elevation located in the exclusive
economic zone of the Philippines.
9
(b) China has aggravated the Parties’ dispute concerning the protection and preservation of the marine
environment at Mischief Reef by inflicting permanent, irreparable harm to the coral reef habitat of that
feature.
(c) China has extended the Parties’ dispute concerning the protection and preservation of the marine
environment by commencing large-scale island-building and construction works at Cuarteron Reef, Fiery
Cross Reef, Gaven Reef (North), Johnson Reef, Hughes Reef, and Subi Reef.
(d) China has aggravated the Parties’ dispute concerning the status of maritime features in the Spratly
Islands and their capacity to generate entitlements to maritime zones by permanently destroying evidence
of the natural condition of Mischief Reef, Cuarteron Reef, Fiery Cross Reef, Gaven Reef (North), Johnson
Reef, Hughes Reef, and Subi Reef.

7.The Tribunal considers it beyond dispute that both Parties are obliged to comply with the Convention,
including its provisions regarding the resolution of disputes, and to respect the rights and freedoms of other
States under the Convention. Neither Party contests this, and the Tribunal is therefore not persuaded that it
is necessary or appropriate for it to make any further declaration.

8. To prove that RP had rights to areas , it must be able to convince the


tribunal to declare the status of maritime features as “rocks” or “low tide
elevations” and not “islands.”

10
5. Chavez v. PEA & AMARI, GR 133250, Nov. 11, 2003 [See #3 Case Set]
NACHURA: [460]
4. Classification of Lands of the Public Domain.
b) Private corporations or associations may not hold such alienable lands of the public
domain except by lease.
i) The 1987 Constitution prohibits private corporations from acquiring alienable lands of the public
domain. Amari, being a private corporation, is barred from such acquisition. The Public
Estates Authority (PEA) is not an end user agency with respect to the reclaimed lands under
the amended Joint Venture Agreement, and PEA may simply turn around and transfer several
hundreds of hectares to a single private corporation in one transaction [Chavez v. Public Estates
Authority, G.R. No. 133250, November 11, 2003].

FACTS:
From the time of Marcos until Estrada, portions of Manila Bay were being reclaimed. A law was passed
creating the Public Estate Authority which was granted with the power to transfer reclaimed lands. Now in
this case, PEA entered into a Joint Venture Agreement with AMARI, a private corporation. Under the Joint
Venture Agreement between AMARI and PEA, several hectares of reclaimed lands comprising the Freedom
Islands and several portions of submerged areas of Manila Bay were going to be transferred to AMARI .

ISSUE:
Whether or not the stipulations in the Amended JVA for the transfer to AMARI of lands, reclaimed or to be
reclaimed, violate the Constitution

RULING: YES

Under the Public Land Act (CA 141, as amended), reclaimed lands are classified as alienable and disposable
lands of the public domain Section 3 of the Constitution:
Alienable lands of the public domain shall be limited to agricultural lands. Private corporations or
associations may not hold such alienable lands of the public domain except by lease.
The 157.84 hectares of reclaimed lands comprising the Freedom Islands, now covered by certificates of title
in the name of PEA, are alienable lands of the public domain. PEA may lease these lands to private
corporations but may not sell or transfer ownership of these lands to private corporations. PEA may
only sell these lands to Philippine citizens, subject to the ownership limitations in the 1987 Constitution and
existing laws.
Clearly, the Amended JVA violates glaringly Sections 2 and 3, Article XII of the 1987 Constitution.
Under Article 1409 of the Civil Code, contracts whose “object or purpose is contrary to law,” or whose
“object is outside the commerce of men,” are “inexistent and void from the beginning.” The Court must
perform its duty to defend and uphold the Constitution, and therefore declares the Amended JVA null and
void ab initio.

We find the cited Ponce Cases inapplicable to the instant case.


First, as Justice Bellosillo himself states in his The correct formulation, however, is that submerged lands are
supplement to his dissent, the Ponce Cases admit owned by the State and are inalienable. Section 2, Article XII
that submerged lands still belong to the National of the 1987 Constitution provides:
Government All lands of the public domain, waters,
minerals, coal, petroleum, and other mineral
oils, all forces of potential energy, fisheries,
forests or timber, wildlife, flora and fauna,
and other natural resources are owned by the
State. With the exception of agricultural
lands, all other natural resources shall not
be alienated. x x x. (Emphasis supplied)
Submerged lands, like the waters (sea or bay) above them, are
part of the States inalienable natural resources. Submerged
lands are property of public dominion, absolutely inalienable
and outside the commerce of man.[10] This is also true with
respect to foreshore lands. Any sale of submerged or foreshore
lands is void being contrary to the Constitution
This is why the Cebu City ordinance merely In the instant case, the bulk of the lands subject of the Amended
granted Essel, Inc. an irrevocable option to purchase JVA are still submerged lands even to this very day, and
the foreshore lands after the reclamation and did not therefore inalienable and outside the commerce of man. Of
actually sell to Essel, Inc. the still to be reclaimed the 750 hectares subject of the Amended JVA, 592.15 hectares
foreshore lands. Clearly, in the Ponce Cases the or 78% of the total area are still submerged, permanently In
option to purchase referred to reclaimed lands, and short, under the Amended JVA the PEA contributed its rights,
11
not to foreshore lands which are privileges and ownership over the Reclamation Area to the
inalienable. Reclaimed lands are no longer foreshore Joint Venture which is 70% owned by Amari. Moreover, the
or submerged lands, and thus may qualify as PEA delegated to Amari the right and privilege to reclaim the
alienable agricultural lands of the public domain submerged lands.under the waters of Manila Bay.
provided the requirements of public land laws are
met.

In the Ponce Cases, the City of Cebu retained . In sharp contrast, in the instant case ownership of the
ownership of the reclaimed foreshore lands reclamation area, including the submerged lands, was
and Essel, Inc. only had an irrevocable option to immediately transferred to the joint
purchase portions of the foreshore lands once venture. Amari immediately acquired the absolute right to own
actually reclaimed 70% percent of the reclamation area, with the deeds of transfer
to be documented and the certificates of title to be issued upon
actual reclamation. Amaris right to own the submerged lands is
immediately effective upon the approval of the Amended JVA
and not merely an option to be exercised in the future if and
when the reclamation is actually realized
Second, in the Ponce Cases the Cebu City ordinance With the subsequent enactment of the Government Auditing
granted Essel, Inc. an irrevocable option to purchase Code (Presidential Decree No. 1445) on 11 June 1978, any sale
from Cebu City not more than 70% of the reclaimed of government land must be made only through public
lands. The ownership of the reclaimed lands bidding. Thus, such an irrevocable option to purchase
remained with Cebu City until Essel, Inc. exercised government land would now be void being contrary to the
its option to purchase requirement of public bidding expressly required in Section
79[17] of PD No. 1445. This requirement of public bidding is
reiterated in Section 379[18] of the 1991 Local Government
Code.[19] Obviously, the ingenious reclamation scheme adopted
in the Cebu City ordinance can no longer be followed in view
of the requirement of public bidding in the sale of government
lands. In the instant case, the Amended JVA is a negotiated
contract which clearly contravenes Section 79 of PD No. 1445.
Third, Republic Act No. 1899 authorized The instant case involves principally submerged lands
municipalities and chartered cities to within Manila Bay. On this score, the Ponce Cases, which were
reclaim foreshore lands. decided based on RA No. 1899, are not applicable to the instant
As we held in the 1998 case of Republic Real Estate case.
Corporation v. Court of Appeals,[20] citing
the Ponce Cases, RA No. 1899 applies only to
foreshore lands, not to submerged lands. In his
concurring opinion in Republic Real Estate
Corporation,
Fourth, the Ponce Cases involve the authority of the However, in the instant case the PEA is not an end user agency
City of Cebu to reclaim foreshore areas pursuant to a with respect to the reclaimed lands under the Amended JVA.
general law, RA No. 1899. The City of Cebu is a
public corporation and is qualified, under the 1935,
1973, and 1987 Constitutions, to hold alienable or
even inalienable lands of the public domain. There is
no dispute that a public corporation is not covered by
the constitutional ban on acquisition of alienable
public lands. Both the 9 July 2002 Decision and the 6
May 2003Resolution of this Court in the instant case
expressly recognize this.

Finally, the Ponce Cases were decided under the However, the 1973 Constitution prohibited private corporations
1935 Constitution which allowed private from acquiring alienable lands of the public domain, and the
corporations to acquire alienable lands of the public 1987 Constitution reiterated this prohibition. Obviously, the
domain. Ponce Cases cannot serve as authority for a private corporation
to acquire alienable public lands, much less submerged lands,
since under the present Constitution a private corporation
like Amari is barred from acquiring alienable lands of the
public domain.
Clearly, the facts in the Ponce Cases are different from the facts
in the instant case.
Moreover, the governing constitutional and statutory
provisions have changed since the Ponce Cases were disposed
of in 1965 and 1966 through minute Resolutions of a divided (6
to 5) Court.
This Resolution does not prejudice any innocent third party
purchaser of the reclaimed lands covered by the Amended
JVA. Neither the PEA nor Amari has sold any portion of the
12
reclaimed lands to third parties. Title to the reclaimed lands
remains with the PEA

As we held in our 9 July 2002 Decision, the Amended JVA violates glaringly Sections 2 and 3, Article XII
of the 1987 Constitution. In our 6 May 2003 Resolution, we DENIED with FINALITY respondents
Motions for Reconsideration. Litigations must end sometime. It is now time to write finis to
this Grandmother of All Scams.

Facts: On February 4, 1977, then President Ferdinand E. Marcos issued Presidential Decree No. 1084
creating PEA. PD No. 1084 tasked PEA "to reclaim land, including foreshore and submerged areas," and "to
develop, improve, acquire, lease and sell any and all kinds of lands." On the same date, then President
Marcos issued Presidential Decree No. 1085 transferring to PEA the "lands reclaimed in the foreshore and
offshore of the Manila Bay" under the Manila-Cavite Coastal Road and Reclamation Project (MCCRRP).

On January 19, 1988, then President Corazon C. Aquino issued Special Patent No. 3517, granting and
transferring to PEA "the parcels of land so reclaimed under the Manila-Cavite Coastal Road and Reclamation
Project (MCCRRP) containing a total area of one million nine hundred fifteen thousand eight hundred ninety
four (1,915,894) square meters." Subsequently, on April 9, 1988, the Register of Deeds of the Municipality
of Parañaque issued Transfer Certificates of Title Nos. 7309, 7311, and 7312, in the name of PEA, covering
the three reclaimed islands known as the "Freedom Islands" located at the southern portion of the Manila-
Cavite Coastal Road, Parañaque City.

PEA and AMARI entered into the JVA through negotiation without public bidding. On April 28, 1995,
the Board of Directors of PEA, in its Resolution No. 1245, confirmed the JVA. On June 8, 1995, then
President Fidel V. Ramos, through then Executive Secretary Ruben Torres, approved the JVA.

The Senate Committees reported the results of their investigation in Senate Committee Report No. 560
dated September 16, 1997. Among the conclusions of their report are: (1) the reclaimed lands PEA seeks to
transfer to AMARI under the JVA are lands of the public domain which the government has not
classified as alienable lands and therefore PEA cannot alienate these lands; (2) the certificates of title
covering the Freedom Islands are thus void, and (3) the JVA itself is illegal.

On December 5, 1997, then President Fidel V. Ramos issued Presidential Administrative Order No. 365
creating a Legal Task Force to conduct a study on the legality of the JVA in view of Senate Committee
Report No. 560. The members of the Legal Task Force were the Secretary of Justice, the Chief Presidential
Legal Counsel, and the Government Corporate Counsel. The Legal Task Force upheld the legality of the
JVA, contrary to the conclusions reached by the Senate Committees.

On April 27, 1998, petitioner Frank I. Chavez ("Petitioner" for brevity) as a taxpayer, filed the instant
Petition for Mandamus with Prayer for the Issuance of a Writ of Preliminary Injunction and Temporary
Restraining Order. Petitioner contends the government stands to lose billions of pesos in the sale by PEA of
the reclaimed lands to AMARI. Petitioner prays that PEA publicly disclose the terms of any renegotiation of
the JVA, invoking Section 28, Article II, and Section 7, Article III, of the 1987 Constitution on the right of
the people to information on matters of public concern.

Due to the approval of the Amended JVA by the Office of the President, petitioner now prays that on
"constitutional and statutory grounds the renegotiated contract be declared null and void."
Issue: The issues raised by petitioner, PEA and AMARI are as follows:
1. Whether the reliefs prayed for are moot and academic because of subsequent events;
2. Whether the petition should be dismissed for failing to observe the principle of governing the heirarchy
of courts;
3. Whether the petition should be dismissed for non-exhaustion of administrative remedies;
4. Whether petitioner has locus standi;
5. Whether the constitutional right to information includes information on on-going neogtiations
BEFORE a final agreement;
6. Whether the stipulations in the amended joint venture agreement for the transfer to AMARI of certain
lands, reclaimed and still to be reclaimed violate the 1987 Constitution; and
7. Whether the Court has jurisdiction over the issue
13
8.whether the amended JVA is grossly disadvantageous to the government

Held: 1. We rule that the signing and of the Amended JVA by PEA and AMARI and its approval by the
President cannot operate to moot the petition and divest the Court of its jurisdiction.

PEA and AMARI have still to implement the Amended JVA. The prayer to enjoin the signing of the
Amended JVA on constitutional grounds necessarily includes preventing its implementation if in the
meantime PEA and AMARI have signed one in violation of the Constitution. Petitioner's principal basis in
assailing the renegotiation of the JVA is its violation of the Section 3, Article XII of the Constitution, which
prohibits the government from alienating lands of the public domain to private corporations. The Amended
JVA is not an ordinary commercial contract but one which seeks to transfer title and ownership to 367.5
hectares of reclaimed lands and submerged areas of Manila Bay to a single private corporation.

Also, the instant petition is a case of first impression being a wholly government owned corporation
performing public as well as proprietary functions. All previous decisions of the Court involving Section 3,
Article XII of the 1987 Constitution, or its counterpart provision in the 1973 Constitution, covered
agricultural lands sold to private corporations which acquired the lands from private parties.

Lastly, there is a need to resolve immediately the constitutional issue raised in this petition because of the
possible transfer at any time by PEA to AMARI of title and ownership to portions of the reclaimed lands.
Under the Amended JVA, PEA is obligated to transfer to AMARI the latter's seventy percent proportionate
share in the reclaimed areas as the reclamation progresses, The Amended JVA even allows AMARI to
mortgage at any time the entire reclaimed area to raise financing for the reclamation project.

2. The instant case, however, raises constitutional issues of transcendental importance to the public. The
Court can resolve this case without determining any factual issue related to the case. Also, the instant case is
a petition for mandamus which falls under the original jurisdiction of the Court under Section 5, Article VIII
of the Constitution. We resolve to exercise primary jurisdiction over the instant case.

3. PEA was under a positive legal duty to disclose to the public the terms and conditions for the sale of its
lands. The law obligated PEA make this public disclosure even without demand from petitioner or from
anyone. PEA failed to make this public disclosure because the original JVA, like the Amended JVA, was the
result of a negotiated contract, not of a public bidding. Considering that PEA had an affirmative statutory
duty to make the public disclosure, and was even in breach of this legal duty, petitioner had the right to seek
direct judicial intervention.

The principle of exhaustion of administrative remedies does not apply when the issue involved is a purely
legal or constitutional question. The principal issue in the instant case is the capacity of AMARI to acquire
lands held by PEA in view of the constitutional ban prohibiting the alienation of lands of the public domain
to private corporations. We rule that the principle of exhaustion of administrative remedies does not apply in
the instant case.

The petitioner has standing to bring this taxpayer's suit because the petition seeks to compel PEA to
comply with its constitutional duties. There are two constitutional issues involved here. First is the right of
citizens to information on matters of public concern. Second is the application of a constitutional provision
intended to insure the equitable distribution of alienable lands of the public domain among Filipino Citizens.
The thrust of the second issue is to prevent PEA from alienating hundreds of hectares of alienable lands of
the public domain in violation of the Constitution, compelling PEA to comply with a constitutional duty to
the nation.

4. Ordinary taxpayers have a right to initiate and prosecute actions questioning the validity of acts or
orders of government agencies or instrumentalities, if the issues raised are of 'paramount public interest,' and
if they 'immediately affect the social, economic and moral well being of the people.'

14
We rule that since the instant petition, brought by a citizen, involves the enforcement of constitutional
rights — to information and to the equitable diffusion of natural resources — matters of transcendental
public importance, the petitioner has the requisite locus standi.

5. The State policy of full transparency in all transactions involving public interest reinforces the people's
right to information on matters of public concern. This State policy is expressed in Section 28, Article II of
the Constitution, thus: “Subject to reasonable conditions prescribed by law, the State adopts and implements
a policy of full public disclosure of all its transactions involving public interest."

Contrary to AMARI's contention, the commissioners of the 1986 Constitutional Commission understood
that the right to information "contemplates inclusion of negotiations leading to the consummation of the
transaction." Certainly, a consummated contract is not a requirement for the exercise of the right to
information. Otherwise, the people can never exercise the right if no contract is consummated, and if one is
consummated, it may be too late for the public to expose its defects.

Requiring a consummated contract will keep the public in the dark until the contract, which may be
grossly disadvantageous to the government or even illegal, becomes a fait accompli.

However, the right to information does not compel PEA to prepare lists, abstracts, summaries and the like
relating to the renegotiation of the JVA. 34 The right only affords access to records, documents and papers,
which means the opportunity to inspect and copy them. One who exercises the right must copy the
records, documents and papers at his expense. The exercise of the right is also subject to reasonable
regulations to protect the integrity of the public records and to minimize disruption to government
operations, like rules specifying when and how to conduct the inspection and copying.

6. Article 339 of the Civil Code of 1889 defined property of public dominion as follows:
"Art. 339. Property of public dominion is —
1. That devoted to public use, such as roads, canals, rivers, torrents, ports and bridges constructed by the
State, riverbanks, shores, roadsteads, and that of a similar character;
2. That belonging exclusively to the State which, without being of general public use, is employed in
some public service, or in the development of the national wealth, such as walls, fortresses, and other works
for the defense of the territory, and mines, until granted to private individuals.

Property devoted to public use referred to property open for use by the public. In contrast, property
devoted to public service referred to property used for some specific public service and open only to those
authorized to use the property. Property of public dominion referred not only to property devoted to public
use, but also to property not so used but employed to develop the national wealth. This class of property
constituted property of public dominion although employed for some economic or commercial activity to
increase the national wealth.

"Art. 341. Property of public dominion, when no longer devoted to public use or to the defense of the
territory, shall become a part of the private property of the State." This provision, however, was not self-
executing. The legislature, or the executive department pursuant to law, must declare the property no longer
needed for public use or territorial defense before the government could lease or alienate the property to
private parties.

Act No. 2874 of the Philippine Legislature


Sec. 55. Any tract of land of the public domain which, being neither timber nor mineral land, shall be
classified as suitable for residential purposes or for commercial, industrial, or other productive purposes
other than agricultural purposes, and shall be open to disposition or concession, shall be disposed of under
the provisions of this chapter, and not otherwise.

The rationale behind this State policy is obvious. Government reclaimed, foreshore and marshy public
lands for non-agricultural purposes retain their inherent potential as areas for public service. This is the
15
reason the government prohibited the sale, and only allowed the lease, of these lands to private parties. The
State always reserved these lands for some future public service.

However, government reclaimed and marshy lands, although subject to classification as disposable
public agricultural lands, could only be leased and not sold to private parties because of Act No. 2874.

The 1987 Constitution continues the State policy in the 1973 Constitution banning private corporations
from acquiring any kind of alienable land of the public domain. Like the 1973 Constitution, the 1987
Constitution allows private corporations to hold alienable lands of the public domain only through lease. As
in the 1935 and 1973 Constitutions, the general law governing the lease to private corporations of reclaimed,
foreshore and marshy alienable lands of the public domain is still CA No. 141.

Without the constitutional ban, individuals who already acquired the maximum area of alienable lands of
the public domain could easily set up corporations to acquire more alienable public lands. An individual
could own as many corporations as his means would allow him. An individual could even hide his ownership
of a corporation by putting his nominees as stockholders of the corporation. The corporation is a convenient
vehicle to circumvent the constitutional limitation on acquisition by individuals of alienable lands of the
public domain.

PD No. 1085, coupled with President Aquino's actual issuance of a special patent covering the Freedom
Islands, is equivalent to an official proclamation classifying the Freedom Islands as alienable or disposable
lands of the public domain. Being neither timber, mineral, nor national park lands, the reclaimed
Freedom Islands necessarily fall under the classification of agricultural lands of the public domain. Under the
1987 Constitution, agricultural lands of the public domain are the only natural resources that the State may
alienate to qualified private parties. All other natural resources, such as the seas or bays, are "waters . . .
owned by the State" forming part of the public domain, and are inalienable pursuant to Section 2, Article XII
of the 1987 Constitution.

In short, DENR is vested with the power to authorize the reclamation of areas under water, while PEA is
vested with the power to undertake the physical reclamation of areas under water whether directly or through
private contractors. DENR is also empowered to classify lands of the public domain into alienable or
disposable lands subject to the approval of the President. On the other hand, PEA is tasked to develop, sell or
lease the reclaimed alienable lands of the public domain.

Clearly, the mere physical act of reclamation by PEA of foreshore or submerged areas does not
make the reclaimed lands alienable or disposable lands of the public domain, much less patrimonial
lands of PEA. Likewise, the mere transfer by the National Government of lands of the public domain to
PEA does not make the lands alienable or disposable lands of the public domain, much less patrimonial lands
of PEA.

There is no express authority under either PD No. 1085 or EO No. 525 for PEA to sell its reclaimed
lands. PD No. 1085 merely transferred "ownership and administration" of lands reclaimed from Manila Bay
to PEA, while EO No. 525 declared that lands reclaimed by PEA "shall belong to or be owned by PEA."
PEA's charter, however, expressly tasks PEA "to develop, improve, acquire, administer, deal in, subdivide,
dispose, lease and sell any and all kinds of lands . . . owned, managed, controlled and/or operated by the
government." 87 (Emphasis supplied) There is, therefore, legislative authority granted to PEA to sell its
lands, whether patrimonial or alienable lands of the public domain. PEA may sell to private parties its
patrimonial properties in accordance with the PEA charter free from constitutional limitations. The
constitutional ban on private corporations from acquiring alienable lands of the public domain does not apply
to the sale of PEA's patrimonial lands.

Moreover, under Section 79 of PD No. 1445, otherwise known as the Government Auditing Code, the
government is required to sell valuable government property through public bidding. Section 79 of PD No.
1445 mandates that:... "In the event that the public auction fails, the property may be sold at a private sale at
16
such price as may be fixed by the same committee or body concerned and approved by the Commission."

However, the original JVA dated April 25, 1995 covered not only the Freedom Islands and the additional
250 hectares still to be reclaimed, it also granted an option to AMARI to reclaim another 350 hectares. The
original JVA, a negotiated contract, enlarged the reclamation area to 750 hectares. The failure of public
bidding on December 10, 1991, involving only 407.84 hectares, is not a valid justification for a negotiated
sale of 750 hectares, almost double the area publicly auctioned.

Jurisprudence holding that upon the grant of the patent or issuance of the certificate of title the alienable
land of the public domain automatically becomes private land cannot apply to government units and entities
like PEA.

The grant of legislative authority to sell public lands in accordance with Section 60 of CA No. 141
does not automatically convert alienable lands of the public domain into private or patrimonial lands.
The alienable lands of the public domain must be transferred to qualified private parties, or to government
entities not tasked to dispose of public lands, before these lands can become private or patrimonial lands.
Otherwise, the constitutional ban will become illusory if Congress can declare lands of the public domain as
private or patrimonial lands in the hands of a government agency tasked to dispose of public lands.

To allow vast areas of reclaimed lands of the public domain to be transferred to PEA as private lands will
sanction a gross violation of the constitutional ban on private corporations from acquiring any kind of
alienable land of the public domain. This scheme can even be applied to alienable agricultural lands of the
public domain since PEA can "acquire . . . any and all kinds of lands."

The 157.84 hectares of reclaimed lands comprising the Freedom Islands, now covered by certificates of
title in the name of PEA, are alienable lands of the public domain. PEA may lease these lands to private
corporations but may not sell or transfer ownership of these lands to private corporations.

7. Considering that the Amended JVA is null and void ab initio, there is no necessity to rule on this last
issue. Besides, the Court is not the trier of facts, and this last issue involves a determination of factual
matters.

WHEREFORE, the petition is GRANTED. The Public Estates Authority and Amari Coastal Bay
Development Corporation are PERMANENTLY ENJOINED from implementing the Amended Joint
Venture Agreement which is hereby declared NULL and VOID ab initio.

17
6. Leonidas v Vargas G.R. No. 201031 December 14, 2017

18
19
20
21
22
23
24
25
7. Cruz v. DENR Secretary, G.R. No. 135385, December 6, 2000
(see Case Set #4)
FACTS:
Petitioners Isagani Cruz and Cesar Europa filed a suit for prohibition and mandamus as citizens and
taxpayers, assailing the constitutionality of certain provisions of Republic Act No. 8371, otherwise known as
the Indigenous People’s Rights Act of 1997 (IPRA) and its implementing rules and regulations (IRR). The
petitioners assail certain provisions of the IPRA and its IRR on the ground that these amount to an
unlawful deprivation of the State’s ownership over lands of the public domain as well as minerals and
other natural resources therein, in violation of the regalian doctrine embodied in section 2, Article XII of
the Constitution.
On March 22, 1999, the Commission on Human Rights (CHR) likewise filed a Motion to Intervene and/or to
Appear as Amicus Curiae. The CHR asserts that IPRA is an expression of the principle of parens patriae and
that the State has the responsibility to protect and guarantee the rights of those who are at a serious
disadvantage like indigenous peoples. For this reason it prays that the petition be dismissed.
On March 23, 1999, another group, composed of the Ikalahan Indigenous People and the Haribon
Foundation for the Conservation of Natural Resources, Inc. (Haribon, et al.), filed a motion to Intervene with
attached Comment-in-Intervention. They agree with the NCIP and Flavier, et al. that IPRA is consistent with
the Constitution and pray that the petition for prohibition and mandamus be dismissed.
The motions for intervention of the aforesaid groups and organizations were granted.
Oral arguments were heard on April 13, 1999. Thereafter, the parties and intervenors filed their respective
memoranda in which they reiterate the arguments adduced in their earlier pleadings and during the hearing.
Petitioners assail the constitutionality of the following provisions of the IPRA and its Implementing Rules
on the ground that they amount to an unlawful deprivation of the States ownership over lands of the
public domain as well as minerals and other natural resources therein, in violation of the regalian
doctrine embodied in Section 2, Article XII of the Constitution:

ISSUE:
Whether or not the IPRA law is unconstitutional.
HELD:
The Supreme Court deliberated upon the matter. After deliberation they voted and reached a 7-7 vote. They
deliberated again and the same result transpired. Since there was no majority vote, Cruz’s petition was
dismissed and the constitutionality of the IPRA law was sustained. Hence, ancestral domains may include
public domain – somehow against the Regalian doctrine.
(1) Section 3(a) which defines the extent and coverage of ancestral domains, and Section 3(b) which, in turn, defines
ancestral lands;
(2) Section 5, in relation to section 3(a), which provides that ancestral domains including inalienable public lands,
bodies of water, mineral and other resources found within ancestral domains are private but community property of the
indigenous peoples;
(3) Section 6 in relation to section 3(a) and 3(b) which defines the composition of ancestral domains and ancestral
lands;
(4) Section 7 which recognizes and enumerates the rights of the indigenous peoples over the ancestral domains;
(5) Section 8 which recognizes and enumerates the rights of the indigenous peoples over the ancestral lands;
(6) Section 57 which provides for priority rights of the indigenous peoples in the harvesting, extraction, development or
exploration of minerals and other natural resources within the areas claimed to be their ancestral domains, and the right
to enter into agreements with nonindigenous peoples for the development and utilization of natural resources therein for
a period not exceeding 25 years, renewable for not more than 25 years; and
(7) Section 58 which gives the indigenous peoples the responsibility to maintain, develop, protect and conserve the
ancestral domains and portions thereof which are found to be necessary for critical watersheds, mangroves, wildlife
sanctuaries, wilderness, protected areas, forest cover or reforestation. [2]

Davide, Jr., C.J., Bellosillo, Melo, Quisumbing, Pardo, Buena, Gonzaga-Reyes, Ynares-Santiago, and De
Leon, Jr., JJ., concur.
Puno, Vitug, Kapunan, Mendoza and Panganiban JJ., see separate opinion

 Most important issue: Whether Sections 3(a) and (b), 5, 6, 7, 8, and 57, and 58 of RA 8371/IPRA
and its IRR are unconstitutional for unlawfully depriving the State of its ownership over lands of the
public domain, minerals, and other natural resources therein, violating the regalian doctrine
enshrined in Section 2, Article XII of the Constitution
o Justice Kapunan: NO.
 Said provisions affirming the ownership by indigenous peoples of their ancestral
lands and domains by virtue of native title (definition: lands held in private
ownership since time immemorial) do not diminish the State’s ownership of lands
within the public domain, because said ancestral lands and domains are
26
considered as private land, and never to have been part of the public domain,
following the doctrine laid down in Cariño v. Insular Government.
 Section 3(a) does not confer or recognize any right of ownership over the natural
resources to the ICCs/IPs. Its purpose is definitional and not declarative of a right or
title.
 Section 57 only grants “priority rights” to ICCs/IPs in the utilization of natural
resources and not absolute ownership thereof.
 The State retains full control over the exploration, development and utilization of
natural resources through the imposition of requirements and conditions for the
utilization of natural resources under existing laws, such as the Small-Scale Mining
Act of 1991 and the Philippine Mining Act of 1995. Neither does the grant of said
rights exclude non-indigenous peoples from undertaking the same activities within
the ancestral domains upon authority granted by the proper governmental agency.
o Justice Puno: NO.
 Ancestral lands and ancestral domains are not part of the lands of the public domain.
They are private and belong to the ICCs/IPs. The classification of lands in the public
domain under Section 3, Article XII of the Constitution does not include ancestral
lands nor ancestral domains. The rights of ICCs/IPs to their ancestral domains and
ancestral lands may be acquired in two modes: (1) by native title over both ancestral
lands and domains; or
(2) by torrens title under the Public Land Act and the Land Registration Act with
respect to ancestral lands only. Both modes presume or recognize the land as
private and not public.
 The right of ownership to ancestral domain under Section 7(a) involves “lands,
bodies of water traditionally and actually occupied by ICCs/IPs, sacred places,
traditional hunting and fishing grounds, and all improvements made by them at any
time within the domains”, not “waters, minerals, coal, petroleum, and other mineral
oils, all forces of potential energy, fisheries, forests or timber, wildlife, flora and
fauna, and other natural resources” enumerated in Section 2, Article XII of the
Constitution. Ownership therefore of natural resources remain with the State.
 Small-scale utilization of resources in Section 7(b) is also allowed under paragraph
3, section 2, Article XII of the Constitution.
 Finally, the large-scale utilization of natural resources in Section 57 of RA
8371/IPRA is allowed under paragraphs 1 and 4, section 2, Article XII of the
Constitution since only “priority rights” (which does not necessarily mean
ownership rights) are given to ICCs/IPs.
 However, by including “natural resources”, Section 1, Part II, Rule III of the
Implementing Rules goes beyond Section 7(a) and therefore unconstitutional.
o Justice Panganiban: YES.
 Section 3(a) [whose definition of ancestral domain encompasses natural resources
found therein], and 3(b) [defines ancestral lands as those possessed by ICCs/IPs
since time immemorial] contravene Section 2, Article XII of the Constitution, which
declares that the State owns all lands of the public domain, minerals, and
natural resources – none of which, except agricultural lands, can be alienated.
In addition, mere possession or utilization of land, however long, does not
automatically convert them into private properties.
 IPRA/RA 8371 does not specify limits to ancestral lands and domains.
 IPRA/RA 8371 relinquishes the State’s power under Section 2, Article XII of the
Constitution of full control of natural resources in ancestral lands and ancestral
domains in favor of ICCs/IPs, who may exercise these rights without any time limit.
In addition, they are also given the right to negotiate directly the terms and
conditions for the exploration of natural resources under Section 7(b), a right vested
by the Constitution only to the State.
o Justice Vitug: YES. Sections 7 and 57 go beyond the context of the fundamental law and
virtually amount to an undue delegation, if not an unacceptable abdication, of State authority
over a significant area of the country and its patrimony
 Whether Sections 51 to 53, 59, 52(i), 63, 65, and 66 of RA 8371/IPRA, defining the powers and
jurisdiction of the NCIP and making customary law applicable to the settlement of disputes
involving ancestral domains and ancestral lands, violate the due process clause of the Constitution
o Justice Kapunan: NO. The fact the NCIP is composed wholly of indigenous peoples does
not mean that it is incapable of being impartial. Moreover, the use of customary laws is
sanctioned by paragraph 2, Section 5 of Article XII of the Constitution.
27
o Justice Puno: Not discussed.
o Justice Panganiban: Not discussed. It is best to await specific cases filed by those whose
rights may have been injured by these provisions.
o Justice Vitug: YES, but only on making customary law applicable to the settlement of
disputes involving ancestral domains and ancestral lands. The second paragraph of Section 5
of Article XII of the Constitution allows Congress to provide for the applicability of
customary laws governing property rights or relations in determining the ownership and
extent of ancestral domains. I do not see this statement as saying that Congress may enact a
law that would simply express that customary laws shall govern and end it there. No
discussion on the powers and jurisdiction of the NCIP.
 Whether Rule VII, Part II, Section 1 of the NCIP Administrative Order No. 1, series of 1998, which
provides that the administrative relationship of the NCIP to the Office of the President is
characterized as a lateral but autonomous relationship for purposes of policy and program
coordination, infringes upon the Presidents power of control over executive departments under
Section 17, Article VII of the Constitution
o Justice Kapunan: NO, since said provision as well as Section 40 of the IPRA expressly
places the NCIP under the Office of the President, and therefore under the President’s
control and supervision with respect to its administrative functions. However, insofar as the
decisions of the NCIP in the exercise of its quasi-judicial powers are concerned, the same are
reviewable by the Court of Appeals, like those of the NLRC and the SEC.
o Justice Puno: Not discussed.
o Justice Panganiban: Not discussed. It is best to await specific cases filed by those whose
rights may have been injured by these provisions.
o Justice Vitug: Not discussed.
NOTES:
 Seven Justices voted to dismiss the petitions
o Justice Kapunan filed an opinion, which the Chief Justice and Justices Bellosillo,
Quisumbing, and Santiago join, sustaining the validity of the challenged provisions of R.A.
8371.
o Justice Puno also filed a separate opinion sustaining all challenged provisions of the law
with the exception of Section 1, Part II, Rule III of NCIP Administrative Order No. 1, series
of 1998, the Rules and Regulations Implementing the IPRA, and Section 57 of the IPRA
which he contends should be interpreted as dealing with the large-scale exploitation of
natural resources and should be read in conjunction with Section 2, Article XII of the 1987
Constitution.
o Justice Mendoza voted to dismiss the petition solely on the ground that it does not raise a
justiciable controversy and petitioners do not have standing to question the constitutionality
of R.A. 8371.
 Seven Justices voted to grant the petition
o Justice Panganiban filed a separate opinion expressing the view that Sections 3 (a)(b), 5, 6, 7
(a)(b), 8, and related provisions of R.A. 8371 are unconstitutional. He reserves judgment on
the constitutionality of Sections 58, 59, 65, and 66 of the law, which he believes must await
the filing of specific cases by those whose rights may have been violated by the IPRA.
o Justice Vitug also filed a separate opinion expressing the view that Sections 3(a), 7, and 57
of R.A. 8371 are unconstitutional. Justice Vitug also mentioned that the petitioners have
standing to raise the issue, as it is of transcendental importance.
o Justices Melo, Pardo, Buena, Gonzaga-Reyes, and De Leon join in the separate opinions of
Justices Panganiban and Vitug.

28
8. OH CHO v. DIRECTOR OF LANDS [ GR No. 48321, Aug 31, 1946 ]

Oh Cho, a citizen of the Republic of China, purchased in 1928 from Antonio, Luis and Rafael
Lagdameo a parcel of land located in the residential district of Guinayangan, Tayabas, which has
been in the continuous, public, and adverse possession of their predecessors in interest as far back
as 1830. On June 17, 1940, Oh Cho applied for the registration of said parcel of land. She Director
of lands opposed the application because, among other grounds, the Constitution prohibits aliens
from acquiring public or private agricultural lands.

One of the witnesses for the applicant, on cross-examination, expressly admitted that this land
in question is susceptible of cultivation and may be converted into an orchard or garden. Rodolfo
Tiquia, inspector of the Bureau of lands, testifying as a witness for the government, stated that the
land, notwithstanding the use to which it is actually devoted, is agricultural land in accordance with
an opinion rendered in 1939 by the Secretary of Justice. The pertinent part of said opinion, penned
by Secretary Jose Abad Santos, later Chief Justice of the Supreme Court, is as follows:

"1. Whether or not the phrase 'public agricultural land' in section 1 Article XII, of the
Constitution may be interpreted to include residential, commercial or industrial lots for purposes of
their disposition.

"1. Section 1, Article XII of the Constitution classifies lands of the public domain in the
Philippines into agricultural, timber and mineral. This is the basic classification adopted since
the enactment of the Act of Congress of July 1, 1902, known as the Philippine Bill. At the time of
the adoption of the Constitution of the Philippines, the term 'agricultural public lands' had,
therefore; acquired a technical meaning in our public laws. The Supreme Court of the Philippines
in the leading, case of Mapa vs. Insular Government, 10 Phil., 175, held that the phrase
'agricultural public lands' means those public lands acquired from Spain which are neither
timber nor mineral lands. This definition has been followed by our Supreme Court in many
subsequent cases. (Montano vs. Ins. Gov't 12 Phil., 572, 574; Santiago vs. Ins. Gov't., 12 Phil.,
593; Ibañes de Aldecoa vs. Ins. Gov't., 13 Phil., 159; Ins. Gov't. vs. Aldecoa & Co., 19 Phil., 505;
516; Mercado vs. Collector of Internal Revenue, 32 Phil.; 271, 276; Molina vs. Rafferty, 38 Phil.,
167, 170; Ramos vs. Director of lands, 39 Phil., 175, 181; Jocson vs. Director of Forestry, 39 Phil.,
560, 564; and Ankron vs. Government of the Philippines, 40 Phil., 10, 14.)

"Residential, commercial or industrial lots forming part of the public domain must have to
be included in one or more of these classes. Clearly, they are neither timber nor mineral, of
necessity, therefore, they must be classified as agricultural.

Viewed from another angle, it has been held that in determining whether lands are agricultural
or not, the character of the land is the test (Odell vs. Durant, 62 N. W. 524; Lerch vs. Missoula Brick
& Tile Co., 123 p. 85). In other words, it is the susceptibility of the land to cultivation for
agricultural purposes by ordinary farming methods which determines whether it is; agricultural or
not (State vs. Stewart, 190 p. 129)."

Judge Pedro Magsalin, of the Court of First Instance of Tayabas, rendered a decision on August
15, 1940, overruling the opposition without much explanation and decreeing the registration prayed
for by the applicant. The Director of lands appealed from the decision, and the Solicitor General
appearing for appellant, maintains that the applicant, not being a citizen of the Philippines, is
disqualified to buy or acquire the parcel of land in question and that the purchase made in
1938 is null and void.

"More specifically, in the case of Ibañez de Aldecoa vs. Insular Government, supra the
Supreme Court held that a residential or building lot, forming part of the public domain, is

29
agricultural land, irrespective of the fact that it is not actually used for purposes or agriculture for
the simple reason that it is susceptible of cultivation and may be converted into a rural estate, and
because when a land is not mineral or forestal in Its nature it must necessarily be included
within the classification of agricultural land. Because of the special applicability of the doctrine
laid down in said case, we quote at some length from the decision therein rendered:

"'From the language of the foregoing provisions of the law, it is deduced that, with the
exception of those comprised within the mineral and timber zone, all lands owned by the State or
by the sovereign nation are public in character, and per se alienable and, provided they are
not destined to the use of public in general or reserved by the Government in accordance with
law, they may be acquired by any private or juridical person; and considering their origin and
primitive state and the general uses to which they are accorded, they are called agricultural lands,
urban lands and building lots being included in this classification for the purpose of distinguishing
rural and urban estates from mineral and timber lands; the transformation they may have undergone
is no obstacle to such classification as the possessor thereof may again convert them into rural
estates.' (Ibañez de Al decoa vs. Insular Government 13 Phil., 161, 163, 164, 165 166; emphasis
added.)

"(b) Under the Constitution and Commonwealth Act No. 141 (Public Land Act), the phrase
'public agricultural land' includes land of the public domain suitable for residential purposes.
"Section 1, Article XII of the Constitution, reads as follows:
"'All agricultural timber, and mineral lands of the public domain, waters, minerals, coal,
petroleum, and other mineral oils, all forces of potential energy, and other natural resources of the
Philippines belong to the State, and their disposition, exploitation, development, or utilization shall
be limited to citizens of the Philippines, or to corporations or associations at least sixty per centum
of the capital of which is owned by such citizens, subject to any existing right, grant, lease, or
concession at the time of the inauguration of the Government established under this Constitution.
Natural resources, with the exception of public agricultural land, shall not be alienated * * *.'
(Emphasis added.)

"In view of the fact that more than one year after the adoption of the Constitution the National
Assembly revised the Public land law and passed Commonwealth Act No. 141, which is a
compilation of the laws relative to lands of the public domain and the amendments thereto, the
statute so revised and compiled must necessarily conform to the Constitution.
"'Where the legislature has revised a statute after a Constitution has been adopted, such a
revision is to be regarded as a legislative construction that the statute so revised conforms to the
Constitution." (59 C.J., 1102; emphasis added.)
"By way of illustration, let us suppose that a piece or tract of public land has been classified
pursuant to section 9 of Commonwealth Act No. 141 as residential land. If, by reason of this
classification, it is maintained that said land has ceased to be agricultural public land, it will no
longer be subject to alienation or disposition by reason of the constitutional provision that only
agricultural lands are alienable; and yet such residential lot is alienable under sections 58, 59, and
60 of Commonwealth Act No. 141 to citizens of the Philippines or to corporations or associations
mentioned, in section 1, Article XII of the Constitution. Therefore, the classification of public
agricultural lands into various subdivisions is only for purposes of administration, alienation
or disposition, but it does not destroy the inherent nature of all such lands as public
agricultural lands.

'"Save in oases of hereditary succession, no private agricultural land shall be transferred or


assigned except to individuals, corporations, or associations qualified to acquire or hold lands of the
public domain in the Philippines.'

30
"We conclude, therefore, that the residential lot which the applicant seeks to register in his
name falls within the meaning of private agricultural land as this phrase is used in our Constitution
and, consequently, is not subject to acquisition by foreigners except by hereditary succession.

All the foregoing show why we, having been a member of the Constitutional Convention, agree
with the Solicitor General's position and concur in the result in this case, although we would go as
far as the outright pronouncement that the purchase made by appellee is null and void.

31
9. La Bugal-B’laan Tribal Assn. v. DENR Secretary, GR 127882, Jan. 27, 2004 (see Case Set #4)
NACHURA: [96]
ii) In La Bugal-B’Laan Tribal Association v. Ramos, G.R. No. 127882, December 1, 2004, the
Court restrained itself from intruding into policy matters to allow the President and Congress
maximum discretion in using the mineral resources of our country and in securing the assistance
of foreign groups to eradicate the grinding poverty of our people and answer their cry for
viable employment opportunities in the country. “The Judiciary is loath (unwilling) to interfere
with the due exercise by co-equal branches of government of their official functions”. Let the
development of the mining industry be the responsibility of the political branches of government.
The questioned provisions of R.A. 7942 (Philippine Mining Act of 1995) are not unconstitutional

FACTS:
This petition for prohibition and mandamus challenges the constitutionality of Republic Act No. 7942 (The
Philippine Mining Act of 1995), its implementing rules and regulations and the Financial and Technical
Assistance Agreement (FTAA) dated March 30, 1995 by the government with Western Mining
Corporation(Philippines) Inc. (WMCP).
Accordingly, the FTAA violated the 1987 Constitution in that it is a service contract and is antithetical to
the principle of sovereignty over our natural resources, because they allowed foreign control over the
exploitation of our natural resources, to the prejudice of the Filipino nation.

ISSUE:
What is the proper interpretation of the phrase “Agreements involving Either Technical or Financial
Assistance” contained in paragraph 4, Section 2, Article XII of the Constitution.

HELD:
The Supreme Court upheld the constitutionality of the Philippine Mining Law, its implementing rules and
regulations – insofar as they relate to financial and technical agreements as well as the subject Financial and
Technical Assistance Agreement.
Full control is not anathematic to day-to-day management by the contractor, provided that the State retains
the power to direct overall strategy; and to set aside, reverse or modify plans and actions of the contractor.
The idea of full control is similar to that which is exercised by the board of directors of a private corporation,
the performance of managerial, operational, financial, marketing and other functions may be delegated to
subordinate officers or given to contractual entities, but the board retains full residual control of the business.

Agreements Involving Technical


or Financial Assistance Are
Service Contracts With Safeguards
From the foregoing, we are impelled to conclude that the phrase agreements involving either technical or
financial assistance, referred to in paragraph 4, are in fact service contracts. But unlike those of the 1973
variety, the new ones are between foreign corporations acting as contractors on the one hand; and on
the other, the government as principal or owner of the works. In the new service contracts, the foreign
contractors provide capital, technology and technical know-how, and managerial expertise in the creation
and operation of large-scale mining/extractive enterprises; and the government, through its agencies (DENR,
MGB), actively exercises control and supervision over the entire operation.
Such service contracts may be entered into only with respect to minerals, petroleum and other mineral oils.

Ut Magis Valeat
Quam Pereat
Under the third principle of constitutional construction laid down in Francisco -- ut magis valeat quam
pereat -- every part of the Constitution is to be given effect, and the Constitution is to be read and
understood as a harmonious whole. Thus, full control and supervision by the State must be understood as
one that does not preclude the legitimate exercise of management prerogatives by the foreign
contractor. Before any further discussion, we must stress the primacy and supremacy of the principle of
sovereignty and State control and supervision over all aspects of exploration, development and utilization of
the countrys natural resources, as mandated in the first paragraph of Section 2 of Article XII.

Sufficient Control Over Mining


Operations Vested in the State
by RA 7942 and DAO 96-40
RA 7942 provides for the States control and supervision over mining operations. It establishes the
mechanism of inspection and visitorial rights over mining operations and institute reportorial requirements.
The foregoing gamut of requirements, regulations, restrictions and limitations imposed upon the FTAA
contractor by the statute and regulations easily overturns petitioners contention. The setup under RA 7942
32
and DAO 96-40 hardly relegates the State to the role of a passive regulator dependent on submitted plans and
reports. On the contrary, the government agencies concerned are empowered to approve or disapprove -
- hence, to influence, direct and change -- the various work programs and the corresponding minimum
expenditure commitments for each of the exploration, development and utilization phases of the mining
enterprise.
Once these plans and reports are approved, the contractor is bound to comply with its commitments therein.
Figures for mineral production and sales are regularly monitored and subjected to government review, in
order to ensure that the products and by-products are disposed of at the best prices possible; even copies of
sales agreements have to be submitted to and registered with MGB. And the contractor is mandated to open
its books of accounts and records for scrutiny, so as to enable the State to determine if the government share
has been fully paid.
The State may likewise compel the contractors compliance with mandatory requirements on mine safety,
health and environmental protection, and the use of anti-pollution technology and facilities. Moreover, the
contractor is also obligated to assist in the development of the mining community and to pay royalties to the
indigenous peoples concerned.
In other words, the FTAA contractor is not free to do whatever it pleases and get away with it; on the
contrary, it will have to follow the government line if it wants to stay in the enterprise. Ineluctably
then, RA 7942 and DAO 96-40 vest in the government more than a sufficient degree of control and
supervision over the conduct of mining operations.

Section 3(aq) of RA 7942


Not Unconstitutional
An objection has been expressed that Section 3(aq)[55] of RA 7942 -- which allows a foreign contractor to
apply for and hold an exploration permit -- is unconstitutional. The reasoning is that Section 2 of Article
XII of the Constitution does not allow foreign-owned corporations to undertake mining operations
directly. They may act only as contractors of the State under an FTAA; and the State, as the party directly
undertaking exploitation of its natural resources, must hold through the government all exploration
permits and similar authorizations. Hence, Section 3(aq), in permitting foreign-owned corporations to
hold exploration permits, is unconstitutional.
The objection, however, is not well-founded. While the Constitution mandates the State to exercise full
control and supervision over the exploitation of mineral resources, nowhere does it require the government
to hold all exploration permits and similar authorizations. In fact, there is no prohibition at all against
foreign or local corporations or contractors holding exploration permits. The reason is not hard to see.
In brief, the exploration permit serves a practical and legitimate purpose in that it protects the
interests and preserves the rights of the exploration permit grantee (the would-be contractor) --
foreign or local -- during the period of time that it is spending heavily on exploration works, without
yet being able to earn revenues to recoup any of its investments and expenditures. Minus this permit
and the protection it affords, the exploration works and expenditures may end up benefiting only claim-
jumpers. Such a possibility tends to discourage investors and contractors. Thus, Section 3(aq) of RA 7942
may not be deemed unconstitutional.

AFTER ALL IS SAID AND DONE, it is clear that there is unanimous agreement in the Court upon the key
principle that the State must exercise full control and supervision over the exploration, development and
utilization of mineral resources.
The crux of the controversy is the amount of discretion to be accorded the Executive Department,
particularly the President of the Republic, in respect of negotiations over the terms of FTAAs, particularly
when it comes to the government share of financial benefits from FTAAs. The Court believes that it is not
unconstitutional to allow a wide degree of discretion to the Chief Executive, given the nature and complexity
of such agreements, the humongous amounts of capital and financing required for large-scale mining
operations, the complicated technology needed, and the intricacies of international trade, coupled with the
States need to maintain flexibility in its dealings, in order to preserve and enhance our countrys
competitiveness in world markets.
Verily, under the doctrine of separation of powers and due respect for co-equal and coordinate branches of
government, this Court must restrain itself from intruding into policy matters and must allow the President
and Congress maximum discretion in using the resources of our country and in securing the assistance of
foreign groups to eradicate the grinding poverty of our people and answer their cry for viable employment
opportunities in the country.

33
DOCTRINE:

No right is absolute, and the proper regulation of a profession, calling, business or trade has always
been upheld as a legitimate subject of a valid exercise of the police power by the State particularly
when their conduct affects either the execution of legitimate governmental functions, the preservation of
the State, the public health and welfare and public morals.

NACHURA:
Police Power
b) Lawful Means: The means employed are reasonably necessary for the accomplishment of the
purpose, and not unduly oppressive on individuals.
i) Police power concerns government enactments, which precisely interfere with personal liberty
or property to promote the general welfare or the common good. A thorough review of the facts
and circumstances leading to the issuance of DOLE Order No. 3 (establishing various procedures and
requirements for screening performing artists as a prerequisite to the processing of any contract of
employment by POEA) shows that the assailed order was issued by the Secretary of Labor
pursuant to a valid exercise of the police power [JMM Promotion and Management, Inc. v. Court of
Appeals, 260 SCRA 319].

O. Protection to labor Sec. 18. Art. II: “The State affirms labor as a primary social economic force. It shall
protect the rights of workers and promote their welfare.”] Read also Sec. 8, Art. Ill; Sec. 2(5), Art. IX-B;
Sec. 3, Art. XIII.
1. In JMM Promotion and Management v. Court of Appeals, 260 SCRA 319, the Supreme Court said
that obviously, protection to labor does not indicate promotion of employment alone. Under the welfare
and social justice provisions of the Constitution, the promotion of full employment, while desirable,
cannot take a backseat to the government’s constitutional duty “to provide mechanisms for the
protection of our workforce, local or overseas.”

Due Process - 4. Meaning of life, liberty and property


c) Property is anything that can come under the right of ownership and be the subject of
contract. It represents more than the things a person owns; it includes the right to secure, use and
dispose of them [Torraco v. Thompson, 263 U.S. 197].
i) Public office is not property; but one unlawfully ousted from it may institute an action to
recover the same, flowing from the de jure officer’s right to office [Nunez v. Averia, 57 SCRA
726], Indeed, the Court while public office is not property to which one may acquire a vested
right, it is nevertheless a protected right [Bince v. Commission on Electiions, 218 SCRA 782].

“One’s employment, profession or trade or calling is a property right,”

and the wrongful interference therewith is an actionable wrong. Thus, an order of suspension,
without opportunity for hearing, violates property rights [Crespo v. Provincial Board, 160 SCRA 66].

But the proper regulation of employment, profession or trade or


calling has been upheld as a legitimate subject of the police power of
the State,
particularly when its conduct affects either the execution of legitimate governmental functions, the
preservation of the State, the public health and welfare, and public morals [JMM Promotion and
Management v. Court of Appeals, supra.].

Equal Protection of Laws – Valid Classifications – Substantial Distinctions


In JMM Promotion and Management v. Court of Appeals, supra., the Court upheld the classification on
the ground that the DOLE Order applies to all performing artists and entertainers destined for jobs
abroad, as they are prone to exploitation and abuse being beyond the physical reach of government
regulatory agencies.

Labor: In JMM Promotion and Management v. Court of Appeals, 260 SCRA 319, the Supreme Court said
that obviously, protection to labor does not mean promotion of employment alone.

FACTS:

Following the much-publicized death of Maricris Sioson in 1991, former President Corazon C. Aquino
ordered a total ban against the deployment of performing artists to Japan and other foreign destinations.
The ban was, however, rescinded after leaders of the overseas employment industry promised to extend full
support for a program aimed at removing kinks in the system of deployment. In its place, the government,
through the Secretary of Labor and Employment, subsequently issued Department Order No. 28 creating
the Entertainment Industry Advisory Council (EIAC) which was tasked with issuing guidelines on the
34
training, testing certification and deployment of performing artists abroad.

Pursuant to the EIAC's recommendations, the Secretary of Labor, on January 6, 1994, issued Department
Order No. 3 establishing various procedures and requirements for screening performing artists under a new
system of training, testing, certification and deployment of the former. Performing artists successfully
hurdling the test, training and certification requirement were to be issued an Artist's Record Book (ARB),
a necessary prerequisite to processing of any contract of employment by the POEA.

Thereafter, the Department of Labor, following the EIAC's recommendation, issued a series of orders fine-
tuning and implementing the new system which include, among others:

Department Order No. 3-B, pertaining to the Artist Record Book (ARB) requirement, which could be
processed only after the artist could show proof of academic and skills training and has passed the required
tests.

Federation of Entertainment Talent Managers of the Philippines (FETMOP), on January 27, 1995 filed a
class suit assailing these department orders, principally contending that said orders 1) violated the
constitutional right to travel; 2) abridged existing contracts for employment; and 3) deprived individual
artists of their licenses without due process of law. FETMOP, likewise, averred that the issuance of the
Artist Record Book (ARB) was discriminatory and illegal and "in gross violation of the constitutional right
... to life liberty and property." Said Federation consequently prayed for the issuance of a writ of preliminary
injunction against the aforestated orders.

JMM Promotion and Management, Inc. And Kary International, Inc., herein petitioners, filed a Motion for
Intervention in said civil case, which was granted by the trial court but which later denied petitioner's prayer
for a writ of preliminary injunction and dismissed the complaint.

ISSUE:

WON the Artist Record Book requirement and the other Department Orders were issued by the Secretary
of Labor pursuant to a valid exercise of the police power.

HELD:
YES. The latin maxim salus populi est suprema lex embodies the character of the entire spectrum of public
laws aimed at promoting the general welfare of the people under the State's police power. As an inherent
attribute of sovereignty which virtually "extends to all public needs," this "least limitable" of
governmental powers grants a wide panoply of instruments through which the state, as parens patriae gives
effect to a host of its regulatory powers.

Thus, police power concerns government enactments which precisely interfere with personal liberty or
property in order to promote the general welfare or the common good. As the assailed Department Order
enjoys a presumed validity, it follows that the burden rests upon petitioners to demonstrate that the said
order, particularly, its ARB requirement, does not enhance the public welfare or was exercised arbitrarily
or unreasonably.

Pursuant to the alarming number of reports that a significant number of Filipina performing artists ended
up as prostitutes abroad (many of whom were beaten, drugged and forced into prostitution), and following
the deaths of a number of these women, the government began instituting measures aimed at
deploying only those individuals who met set standards which would qualify them as legitimate
performing artists. In spite of these measures, however, a number of our countrymen have nonetheless
fallen victim to unscrupulous recruiters, ending up as virtual slaves controlled by foreign crime syndicates
and forced into jobs other than those indicated in their employment contracts. Worse, some of our women
have been forced into prostitution.

Clearly, the welfare of Filipino performing artists, particularly the women was paramount in the issuance
of Department Order No. 3. Short of a total and absolute ban against the deployment of performing artists
to "high-risk" destinations, a measure which would only drive recruitment further underground, the new
scheme at the very least rationalizes the method of screening performing artists by requiring
reasonable educational and artistic skills from them and limits deployment to only those individuals
adequately prepared for the unpredictable demands of employment as artists abroad. It cannot be
35
gainsaid that this scheme at least lessens the room for exploitation by unscrupulous individuals and
agencies.

As to the other provisions of Department Order No. 3 questioned by petitioners, we see nothing wrong with
the requirement for document and booking confirmation (D.O. 3-C), a minimum salary scale (D.O. 3-E),
or the requirement for registration of returning performers. The requirement for a venue certificate or other
documents evidencing the place and nature of work allows the government closer monitoring of foreign
employers and helps keep our entertainers away from prostitution fronts and other worksites associated
with unsavory, immoral, illegal or exploitative practices. Parenthetically, none of these issuances appear to
us, by any stretch of the imagination, even remotely unreasonable or arbitrary. They address a felt need of
according greater protection for an oft-exploited segment of our OCW's. They respond to the industry's
demand for clearer and more practicable rules and guidelines. Many of these provisions were fleshed out
following recommendations by, and after consultations with, the affected sectors and non-government
organizations. On the whole, they are aimed at enhancing the safety and security of entertainers and artists
bound for Japan and other destinations, without stifling the industry's concerns for expansion and growth.

In any event, apart from the State's police power, the Constitution itself mandates government to extend the
fullest protection to our overseas workers. The basic constitutional statement on labor, embodied in Section
18 of Article II of the Constitution provides:
Sec. 18.The State affirms labor as a primary social economic force. It shall protect the rights of workers
and promote their welfare.

Obviously, protection to labor does not indicate promotion of employment alone. Under the welfare and
social justice provisions of the Constitution, the promotion of full employment, while desirable, cannot take
a backseat to the government's constitutional duty to provide mechanisms for the protection of our
workforce, local or overseas.

We now go to petitioners' assertion that the police power cannot, nevertheless, abridge the right of our
performing workers to return to work abroad after having earlier qualified under the old process, because,
having previously been accredited, their accreditation became a "property right," protected by the due
process clause. We find this contention untenable.

A profession, trade or calling is a property right within the meaning of our constitutional guarantees. One
cannot be deprived of the right to work and the right to make a living because these rights are
property rights, the arbitrary and unwarranted deprivation of which normally constitutes an actionable
wrong.

Nevertheless, no right is absolute, and the proper regulation of a profession, calling, business or trade has
always been upheld as a legitimate subject of a valid exercise of the police power by the State particularly
when their conduct affects either the execution of legitimate governmental functions, the preservation of
the State, the public health and welfare and public morals. According to the maxim, sic utere tuo ut alienum
non laedas,1 it must of course be within the legitimate range of legislative action to define the mode and
manner in which every one may so use his own property so as not to pose injury to himself or others.

In any case, where the liberty curtailed affects at most the rights of property, the permissible scope of
regulatory measures is certainly much wider. To pretend that licensing or accreditation requirements
violates the due process clause is to ignore the settled practice, under the mantle of the police power, of
regulating entry to the practice of various trades or professions. Professionals leaving for abroad are
required to pass rigid written and practical exams before they are deemed fit to practice their trade. Seamen
are required to take tests determining their seamanship. Locally, the Professional Regulation Commission
has began to require previously licensed doctors and other professionals to furnish documentary proof that
they had either re-trained or had undertaken continuing education courses as a requirement for renewal of
their licenses. It is not claimed that these requirements pose an unwarranted deprivation of a property right
under the due process clause. So long as professionals and other workers meet reasonable regulatory
standards no such deprivation exists.

1
use your own property in such a manner as not to injure that of another
36
10. J.G. Summit Holdings vs. CA G.R. No. 124293 September 24, 2003

FACTS:

National Investment and Development Corporation (NIDC) and Kawasaki Heavy Industries entered into a
Joint Venture Agreement in a shipyard business named PHILSECO, with a shareholding of 60-40
respectively. NIDC’s interest was later transferred to the National Government.

Pursuant to President Aquino’s Proclamation No.5, which established the Committee on Privatization (COP)
and Asset Privatization Trust (APT), and allowed for the disposition of the government’s non-performing
assets, the latter allowed Kawasaki Heavy Industries to choose a company to which it has stockholdings, to
top the winning bid of JG Summit Holdings over PHILSECO. JG Summit protested alleging that such act
would effectively increase Kawasaki’s interest in PHILSECO—a shipyard is a public utility–and thus
violative of the Constitution.

--
On 27 January 1977, the National Investment and Development Corporation (NIDC), a government
corporation, entered into a Joint Venture Agreement (JVA) with Kawasaki Heavy Industries, Ltd. of Kobe,
Japan (Kawasaki) for the construction, operation, and management of the Subic National Shipyard, Inc.
(SNS), which subsequently became the Philippine Shipyard and Engineering Corporation (PHILSECO).
Under the JVA, NIDC and Kawasaki would maintain a shareholding proportion of 60% - 40%,
respectively. One of the provisions of the JVA accorded the parties the right of first refusal should either
party sell, assign or transfer its interest in the joint venture. On 25 November 1986, NIDC transferred all its
rights, title and interest in PHILSECO to the Philippine National Bank (PNB).

More than two months later or on 3 February 1987, by virtue of Administrative Order 14, PNB's interest in
PHILSECO was transferred to the National Government. Meanwhile, on 8 December 1986, President
Corazon C. Aquino issued Proclamation 50 establishing the Committee on Privatization (COP) and the Asset
Privatization Trust (APT) to take title to and possession of, conserve, manage and dispose of non-performing
assets of the National Government. On 27 February 1987, a trust agreement was entered into between the
National Government and the APT by virtue of which the latter was named the trustee of the National
Government's share in PHILSECO. In 1989, as a result of a quasi-reorganization of PHILSECO to settle its
huge obligations to PNB, the National Government's shareholdings in PHILSECO increased to 97.41%
thereby reducing Kawasaki's shareholdings to 2.59%. Exercising their discretion, the COP and the APT
deemed it in the best interest of the national economy and the government to privatize PHILSECO by selling
87.67% of its total outstanding capital stock to private entities.

After a series of negotiations between the APT and Kasawaki, they agreed that the latter's right of first
refusal under the JVA be "exchanged" for the right to top by 5% the highest bid for said shares. They further
agreed that Kawasaki would be entitled to name a company in which it was a stockholder, which could
exercise the right to top. On 7 September 1990, Kawasaki informed APT that Philyards Holdings, Inc. (PHI)
would exercise its right to top by 5%. At the pre-bidding conference held on 28 September 1993, interested
bidders were given copies of the JVA between NIDC and Kawasaki, and of the Asset Specific Bidding
Rules (ASBR) drafted for the 87.67% equity (sic) in PHILSECO of the National Government.
The provisions of the ASBR were explained to the interested bidders who were notified that bidding would
be held on 2 December 1993. At the public bidding on said date, the consortium composed of JG Summit
Holdings, Inc. (JGSMI), Sembawang Shipyard Ltd. of Singapore (Sembawang), and Jurong Shipyard
Limited of Malaysia (Jurong), was declared the highest bidder at P2.03 billion. The following day, the
COP approved the sale of 87.67% National Government shares of stock in PHILSECO to said consortium. It
notified JGSMI of said approval "subject to the right of Kawasaki Heavy Industries, Inc./Philyards
Holdings, Inc. to top JGSMI's bid by 5% as specified in the bidding rules."

On 29 December 1993, JGSMI informed the APT that it was protesting the offer of PHI to top its bid on the
grounds that:
(a) the Kawasaki/PHI consortium composed of Kawasaki, Philyards, Mitsui, Keppel, SM Group, ICTSI
and Insular Life violated the ASBR because the last four (4) companies were the losing bidders (for P1.528
billion) thereby circumventing the law and prejudicing the weak winning bidder;
(b) only Kawasaki could exercise the right to top;
(c) giving the same option to top to PHI constituted unwarranted benefit to a third party;
(d) no right of first refusal can be exercised in a public bidding or auction sale, and
(e) the JG Summit Consortium was not estopped from questioning the proceedings.
On 2 February 1994, JGSMI was notified that PHI had fully paid the balance of the purchase price of the
subject bidding. On 7 February 1994, the APT notified JGSMI that PHI had exercised its option to top the

37
highest bid and that the COP had approved the same on 6 January 1994. On 24 February 1994, the APT and
PHI executed a Stock Purchase Agreement. Consequently, JGSMI filed with the Supreme Court a petition
for mandamus under GR 114057. On 11 May 1994, said petition was referred to the Court of Appeals. On 18
July 1995, the Court of Appeals "denied" for lack of merit the petition for mandamus. JGSMI filed a motion
for the reconsideration of said Decision which was denied on 15 March 1996. JGSMI filed the petition for
review on certiorari.

ISSUE:
Whether or not respondents’ act is valid.

HELD:
No.
A shipyard such as PHILSECO being a public utility as provided by law, the following provision of the
Article XII of the Constitution applies:

“Sec. 11. No franchise, certificate, or any other form of authorization for the
operation of a public utility shall be granted except to citizens of the Philippines or to
corporations or associations organized under the laws of the Philippines at least sixty
per centum of whose capital is owned by such citizens, nor shall such franchise,
certificate, or authorization be exclusive in character or for a longer period than fifty
years.
Neither shall any such franchise or right be granted except under the condition
that it shall be subject to amendment, alteration, or repeal by the Congress when the
common good so requires. The State shall encourage equity participation in public
utilities by the general public. The participation of foreign investors in the governing
body of any public utility enterprise shall be limited to their proportionate share in its
capital, and all the executive and managing officers of such corporation or
association shall be citizens of the Philippines.”

Notably, paragraph 1.4 of the JVA accorded the parties the right of first refusal “under the same terms.” This
phrase implies that when either party exercises the right of first refusal under paragraph 1.4, they can only
do so to the extent allowed them by paragraphs 1.2 and 1.3 of the JVA or under the proportion of 60%-
40% of the shares of stock.
Thus, should the NIDC opt to sell its shares of stock to a third party, Kawasaki could only exercise its right
of first refusal to the extent that its total shares of stock would not exceed 40% of the entire shares of
stock of SNS or PHILSECO. The NIDC, on the other hand, may purchase even beyond 60% of the total
shares. As a government corporation and necessarily a 100% Filipino-owned corporation, there is nothing to
prevent its purchase of stocks even beyond 60% of the capitalization as the Constitution clearly limits only
foreign capitalization.

The progenitor of this constitutional provision, Article XIV, Section 5 of the 1973 Constitution, required the
same proportion of 60% - 40% capitalization. The JVA between NIDC and Kawasaki entered into on 27
January 1977 manifests the intention of the parties to abide by the constitutional mandate on capitalization of
public utilities. The joint venture created between NIDC and Kawasaki falls within the purview of an
"association" pursuant to Section 5 of Article XIV of the 1973 Constitution and Section 11 of Article XII of
the 1987 Constitution. Consequently, a joint venture that would engage in the business of operating a public
utility, such as a shipyard, must observe the proportion of 60%-40% Filipino-foreign capitalization. Further,
paragraph 1.4 of the JVA accorded the parties the right of first refusal "under the same terms." This phrase
implies that when either party exercises the right of first refusal under paragraph 1.4, they can only do so to
the extent allowed them by paragraphs 1.2 and 1.3 of the JVA or under the proportion of 60%-40% of the
shares of stock. Thus, should the NIDC opt to sell its shares of stock to a third party, Kawasaki could only
exercise its right of first refusal to the extent that its total shares of stock would not exceed 40% of the entire
shares of stock of SNS or PHILSECO. The NIDC, on the other hand, may purchase even beyond 60% of the
total shares. As a government corporation and necessarily a 100% Filipino-owned corporation, there is
nothing to prevent its purchase of stocks even beyond 60% of the capitalization as the Constitution clearly
limits only foreign capitalization. Kawasaki was bound by its contractual obligation under the JVA that
limits its right of first refusal to 40% of the total capitalization of PHILSECO.
Thus, Kawasaki cannot purchase beyond 40% of the capitalization of the joint venture on account of
both constitutional and contractual proscriptions. From the facts on record, it appears that at the outset,
the APT and Kawasaki respected the 60%-40% capitalization proportion in PHILSECO.
However, APT subsequently encouraged Kawasaki to participate in the public bidding of the National
Government's shareholdings of 87.67% of the total PHILSECO shares, definitely over and above the 40%
limit of its shareholdings. In so doing, the APT went beyond the ambit of its authority.

38
11. Francisco Tatad vs Jesus Garcia, Jr. November 27, 2012 243 SCRA 436
– Business Organization – Corporation Law – Corporate Nationality – Public Utility – Nationality
Requirement in Nationalized Areas of Activity

In 1989, the government planned to build a railway transit line along EDSA. No bidding was made but
certain corporations were invited to prequalify. The only corporation to qualify was the EDSA LRT
Consortium which was obviously formed for this particular undertaking. An agreement was then made
between the government, through the Department of Transportation and Communication (DOTC), and
EDSA LRT Consortium.
The agreement was based on the Build-Operate-Transfer scheme provided for by law (RA 6957, amended by
RA 7718). Under the agreement, EDSA LRT Consortium shall build the facilities, i.e., railways, and shall
supply the train cabs. Every phase that is completed shall be turned over to the DOTC and the latter shall pay
rent for the same for 25 years. By the end of 25 years, it was projected that the government shall have fully
paid EDSA LRT Consortium. Thereafter, EDSA LRT Consortium shall sell the facilities to the government
for $1.00.

However, Senators Francisco Tatad, John Osmeña, and Rodolfo Biazon opposed the implementation of said
agreement as they averred that EDSA LRT Consortium is a foreign corporation as it was organized
under Hongkong laws; that as such, it cannot own a public utility such as the EDSA railway transit because
this falls under the nationalized areas of activities. The petition was filed against Jesus Garcia, Jr. in his
capacity as DOTC Secretary.

ISSUE: Whether or not the petition shall prosper.

HELD: No. The Supreme Court made a clarification.


The SC ruled that EDSA LRT Consortium, under the agreement, does not and will not become the
owner of a public utility hence, the question of its nationality is misplaced. It is true that a foreign
corporation cannot own a public utility but in this case what EDSA LRT Consortium will be owning
are the facilities that it will be building for the EDSA railway project. There is no prohibition against a
foreign corporation to own facilities used for a public utility.
Further, it cannot be said that EDSA LRT Consortium will be the one operating the public utility for it will
be DOTC that will operate the railway transit. DOTC will be the one exacting fees from the people for the
use of the railway and from the proceeds, it shall be paying the rent due to EDSA LRT Consortium.
All that EDSA LRT Consortium has to do is to build the facilities and receive rent from the use thereof by
the government for 25 years – it will not operate the railway transit. Although EDSA LRT Consortium is a
corporation formed for the purpose of building a public utility it does not automatically mean that it is
operating a public utility. The moment for determining the requisite Filipino nationality is when the entity
applies for a franchise, certificate or any other form of authorization for that purpose.

39
12. Hulst vs PR. Builders Inc. G.R. No 156364, September 25, 2008

FACTS:
The Petitioner and his spouse, both Dutch Nationals, entered into a Contract to Sell with PR Builders, Inc. to
purchase a 210-sq m residential unit in the respondent's townhouse project in Batanagas. When PR Builder's
failed to comply with their verbal promise to complete the project, the spouses Hulst filed a complaint for
recession of contract with interest, damages and attorney's fees before the Housing and Land Regulatory
Board (HLURB), which then was granted.
A Writ of Execution was then addressed to the Ex-Officio Sheriff of the RTC of Tanauan, Batangas, but
upon the complaint of the respondent, the levy was set aside, leaving only the respondent's personal
properties to be levied first. The Sheriff set a public auction of the said levied properties, however, the
respondent filed a motion to quash Writ of levy on the ground that the sheriff made an over levy since the
aggregate appraised value of the properties at P6,500 per sq m is P83,616,000. Instead of resolving the
objection of the respondent's regarding the auction, the Sheriff proceeded with the auction since there was no
restraining order from the HLURB. The 15 parcels of land was then awarded to Holly Properties Realty at a
bid of P5,450,653. (ang value lang is around 6M to answer for the debt of PR Builders, thus kulang ng
around 77M)
On the same day, the Sheriff remitted the legal fees and submitted to contracts of sale to HLURB, however,
he then received orders to suspend proceedings on the auction for the reason that the market value of the
properties was not fair. There was disparity between the appraised value and the value made by the petitioner
and the Sheriff, which should've been looked into by the Sheriff before making the sale. While an
inadequacy in price is not a ground to annul such sale, the court is justified to such intervention where
the price shocks the conscience.

ISSUE:
1. Whether or not the Sheriff erred in the value that was attached to the properties during the auction and as
well as disregarding the objection made by the respondent's?
2. Whether or not the market value of the said property was inadequate?
2. Whether or not the spouses Hulst's request for damages is actionable?

HELD:
1. No. According to the Rules of Court, the value of the property levied is not required to be exactly the same
as the judgment debt. In the levy of property, the Sheriff does not determine the exact valuation of the levied
property. The Sheriff is left to his own judgment. He should be allowed a reasonable margin between the
value of the property levied upon and the amount of the execution; the fact that the Sheriff levies upon a
little more than is necessary to satisfy the execution does not render his actions improper.

In the absence of a restraining order, no error can be imputed to the Sheriff in proceeding with
the auction sale despite the pending motion to quash the levy filed by the respondents with the
HLURB. Sheriff’s, as officers charged with the task of the enforcement and/or implementation of
judgments, must act with considerable dispatch so as not to unduly delay the administration of justice. It is
not within the jurisdiction of the Sheriff to consider and resolve respondent's objection to the continuation of
the conduct of the auction sale. The Sheriff has no authority, on his own, to suspend the auction sale. His
duty being ministerial, he has no discretion to postpone the conduct of the auction sale.

2. No. The HLURB Arbiter and Director had no sufficient factual basis to determine the value of the
levied property. The Appraisal report, that was submitted, was based on the projected value of the
townhouse project after it shall have been fully developed, that is, on the assumption that the residential
units appraised had already been built. Since it is undisputed that the townhouse project did not push
through, the projected value did not become a reality. Thus, the appraisal value cannot be equated with the
fair market value.

The HLURB Arbiter and Director gravely abused their discretion in setting aside the levy conducted by the
Sheriff for the reason that the auction sale conducted by the sheriff rendered moot and academic the motion
to quash the levy. The HLURB Arbiter lost jurisdiction to act on the motion to quash the levy by virtue
of the consummation of the auction sale. Absent any order from the HLURB suspending the auction sale,
the sheriff rightfully proceeded with the auction sale. The winning bidder had already paid the winning bid.
The legal fees had already been remitted to the HLURB. The judgment award had already been turned over
to the judgment creditor. What was left to be done was only the issuance of the corresponding certificates of
sale to the winning bidder. In fact, only the signature of the HLURB Director for that purpose was needed a
purely ministerial act.

40
3. No. Under Article 12, Sec.7 of the 1987 Constitution, foreign nationals, the spouses Hulst, are disqualified
form owning real property. However, under article 1414 of the Civil Code, one who repudiates the
agreement and demands his money before the illegal act has taken place is entitled to recover.
Petitioner is therefore entitled to recover what he has paid, although the basis of his claim for rescission,
which was granted by the HLURB, was not the fact that he is not allowed to acquire private land under the
Philippine Constitution. But petitioner is entitled to the recovery only of the amount of P3,187,500.00,
representing the purchase price paid to respondent. No damages may be recovered on the basis of a void
contract; being nonexistent, the agreement produces no juridical tie between the parties involved. Further,
petitioner is not entitled to actual as well as interests thereon, moral and exemplary damages and attorney's
fees.

Section 7 of Article XII of the 1987 Constitution provides:

Sec. 7. Save in cases of hereditary succession, no private lands shall be transferred or conveyed except to
individuals, corporations, or associations qualified to acquire or hold lands of the public domain.
(Emphasis supplied).

The capacity to acquire private land is made dependent upon the capacity to acquire or hold lands of the
public domain. Private land may be transferred or conveyed only to individuals or entities qualified to
acquire lands of the public domain. The 1987 Constitution reserved the right to participate in the disposition,
exploitation, development and utilization of lands of the public domain for Filipino citizens[25] or
corporations at least 60 percent of the capital of which is owned by Filipinos. [26] Aliens, whether individuals
or corporations, have been disqualified from acquiring public lands; hence, they have also been disqualified
from acquiring private lands.[27]

Generally, parties to a void agreement cannot expect the aid of the law; the courts leave them as they are,
because they are deemed in pari delicto or in equal fault. This rule, however, is subject to exceptions[32] that
permit the return of that which may have been given under a void contract to: );[34] (c) the party repudiating
the void contract before the illegal purpose is accomplished or before damage is caused to a third
person and if public interest is subserved by allowing recovery (Art. 1414, Civil Code);[35]

It is significant to note that the agreement executed by the parties in this case is a Contract to Sell and not a
contract of sale. In a contract to sell, the prospective seller agrees to transfer ownership of the property to the
buyer upon the happening of an event, which normally is the full payment of the purchase price. But even
upon the fulfillment of the suspensive condition, ownership does not automatically transfer to the buyer. The
prospective seller still has to convey title to the prospective buyer by executing a contract of absolute sale.[42]

Since the contract involved here is a Contract to Sell, ownership has not yet transferred to the petitioner
when he filed the suit for rescission. While the intent to circumvent the constitutional proscription on aliens
owning real property was evident by virtue of the execution of the Contract to Sell, such violation of the law
did not materialize because petitioner caused the rescission of the contract before the execution of the final
deed transferring ownership.

Thus, exception (c) finds application in this case. Under Article 1414, one who repudiates the agreement and
demands his money before the illegal act has taken place is entitled to recover. Petitioner is therefore entitled
to recover what he has paid, although the basis of his claim for rescission, which was granted by the
HLURB, was not the fact that he is not allowed to acquire private land under the Philippine Constitution. But
petitioner is entitled to the recovery only of the amount of P3,187,500.00, representing the purchase price
paid to respondent. No damages may be recovered on the basis of a voidcontract; being nonexistent, the
agreement produces no juridical tie between the parties involved.[43] Further, petitioner is not entitled to
actual as well as interests thereon,[44] moral and exemplary damages and attorneys fees.

41
13. Moss vs. Director of Lands G.R. No. L-27170 November 22, 1977

Eugene Moss appealed from the decision of the Court of First Instance of Leyte, Carigara
Branch VI, denying his application for the registration of a ten-hectare island on the ground that,
being an American citizen or an alien, he is disqualified to acquire lands under section 5, Article
XIII of the 1935 Constitution, as held in Krivenko v. Register of Deeds, 79 Phil. 461 (Land
Registration Case No. N-68, LRC Record No. N-27971).

The Solicitor General, disagreeing with the trial court, recommends in his brief for the Director of
Lands that the decision be reversed and that the application of Moss be granted.

On Carigara Bay there is an islet known as Calumpihan Island within the jurisdiction of Barrio
Calumpihan, Capoocan, Leyte. It is planted to coconuts more than sixty to eighty years old in 1966.
Fruit trees, corn and tobacco are also grown on the land.chanrobles virtual lawlibrary

That land was already declared for tax purposes in 1930. It was then owned by Vicente Pragas who
had possessed and cultivated it since 1894. On November 14, 1932 Pragas sold the island to
Eduardo Cecilio. On March 23, 1937 Cecilio sold it to Catalina Pabilion. Then, on December 23,
1944 the spouses Catalina Pabilion and Guillermo Dadizon sold the land to Rufino M. Pascual who,
in turn, sold it on January 20, 1945 for P1,200 to Eugene Moss and Albert Boyd Cassidy,
American nationals.

In an action to quiet title filed by Moss against Cassidy, an absentee, Moss was adjudged the sole
owner of the and in a decision dated March 27, 1962 rendered by the Court of First Instance of
Leyte in Civil Case No. 645.

Moss declared the land for tax purposes on June 27, 1950. He paid the realty taxes on the said land
for the years 1945 to 1966. He had it surveyed on November 20, 1962.

On April 3, 1965 Moss, through Dr. Teodorico H. Jaceldo, his administrator and attorney-in-fact,
filed an application for the registration of the said land. Moss is a retired army colonel, an American
citizen, married to Eileen Moss, and a former resident of 504 Glen Park Drive, Nashville,
Tennessee and now Los Fresnos, Texas.

As already stated, the trial court denied the application for registration because Moss is an alien.
That holding is erroneous. It is erroneous because while aliens are disqualified to acquire lands
under the 1935 Constitution (Levy Hermanos v. Ledesma, 69 Phil. 49; Krivenko v. Register of
Deeds, supra), citizens of the United States can acquire lands like Filipino citizens.

The trial court overlooked the Ordinance which was appended to the 1935 Constitution by
Resolution No. 39 of the National Assembly dated September 15, 1939. That resolution was
approved by the President of the United States on November 10, 1939. It provides as
follows:chanrobles lawlibrary : rednad

"ORDINANCE APPENDED TO THE CONSTITUTION

"SECTION 1. Notwithstanding the provisions of the foregoing Constitution, pending the final and
complete withdrawal of the sovereignty of the United States over the Philippines —

x x x

"(17) Citizens and corporations of the United States shall enjoy in the Commonwealth of the
Philippines all the civil rights of the citizens and corporations, respectively, thereof." (1 Philippine
Annotated Laws, pp. 31-34)

42
That Ordinance was made a part of the 1935 Constitution as directed in section 2 of the Tydings-
McDuffie Law or the Independence law. Subsection (17) quoted above is a reproduction of
subsection (16) of section 2(a) of the Independence Law.

Moss validly acquired the island in question under the provisions of subsection (17), section 1 of
the aforementioned Ordinance (See sec. 127 of Public Land Law; Republic v. Quasha, L-30299,
August 17, 1972, 46 SCRA 160, 172-173. Note that the instant case is not covered by section 11,
Article XVII of the 1973 Constitution and by Presidential Decree No. 713 dated May 27, 1975, 71
O.G. 4115 re residential lands of American citizens).

The proclamation of Philippine independence on July 4, 1946 did not impair Moss’ proprietary
rights over the said land because the 1935 Constitution provides that upon the proclamation of
Philippine independence "all existing property rights of citizens or corporations of the United
States shall be acknowledged, respected, and safeguarded to the same extent as property rights of
citizens of the Philippines" (Sec. 1[1], Article XVII).

That constitutional provision is implemented in Article VI of the Treaty of General Relations


entered into between the Republic of the Philippines and the United States on July 4, 1946 which
provides that all existing property rights of citizens and corporations of the United States in
the Republic of the Philippines shall be acknowledged, respected and safeguarded to the same
extent as property rights of citizens and corporations of the Republic of the Philippines (42 O.G.
1651. See Republic Act No. 76).

When Moss purchased the land, Leyte was already liberated as shown in the proclamation of
General Douglas MacArthur dated October 23, 1944. (Moss and Cassidy were allegedly members
of the American liberation forces that landed in Leyte). See 41 O.G. 147.

Since Moss and his predecessors in interest have been in possession en concepto de dueño of
Calumpihan Island for more than thirty years immediately preceding the filing of his application for
confirmation of his title, he is entitled to the registration of his title to the island (Sec. 48[b], Public
Land Law, as amended by Republic Act No. 1942).

WHEREFORE, the trial court’s decision is reversed and set aside and the application of Eugene
Moss, of age, citizen of the United States, married to Eillen Moss, P. O. Box 184, Los Fresnos,
Texas, U. S. A., for the confirmation and registration of his title to the land shown in the plan Psu-
198022, is hereby granted.

SO ORDERED.

43
14. Board of Medicine vs Yasukuyi Ota G.R. No. 166907 July 14, 2007

FACTS:
 Yasuyuki Ota (respondent) is a Japanese national, married to a Filipina, who has continuously resided in
the Philippines for more than 10 years. He graduated from Bicol Christian College of Medicine on
April 21, 1991 with a degree of Doctor of Medicine. After successfully completing a one-year post
graduate internship training at the Jose Reyes Memorial Medical Center, he filed an application to take
the medical board examinations in order to obtain a medical license. He was required by the (PRC) to
submit an affidavit of undertaking, stating among others that should he successfully pass the same, he
would not practice medicine until he submits proof that reciprocity exists between Japan and the
Philippines in admitting foreigners into the practice of medicine.
 Respondent submitted a duly notarized English translation of the Medical Practitioners Law of Japan
duly authenticated by the Consul General of the Philippine Embassy to Japan, Jesus I. Yabes; thus, he
was allowed to take the Medical Board Examinations in August 1992, which he subsequently passed.
 In spite of all these, the Board of Medicine (Board) of the PRC, in a letter dated March 8, 1993, denied
respondent's request for a license to practice medicine in the Philippines on the ground that the Board
“believes that no genuine reciprocity can be found in the law of Japan as there is no Filipino or
foreigner who can possibly practice there.”
 Respondent then filed a Petition for Certiorari and Mandamus against the Board before the RTC of
Manila. RTC ruled for the Yasuki.
 The Board and the PRC (petitioners) appealed the case to the CA, stating that while respondent
submitted documents showing that foreigners are allowed to practice medicine in Japan, it was not
shown that the conditions for the practice of medicine there are practical and attainable by a foreign
applicant, hence, reciprocity was not established; also, the power of the PRC and the Board to regulate
and control the practice of medicine is discretionary and not ministerial, hence, not compellable by a writ
of mandamus.
 The CA denied the appeal and affirmed the ruling of the RTC.
ISSUES:
WHETHER THE COURT OF APPEALS COMMITTED A REVERSIBLE ERROR IN FINDING THAT
RESPONDENT HAD ESTABLISHED THE EXISTENCE OF RECIPROCITY IN THE PRACTICE OF
MEDICINE BETWEEN THE PHILIPPINES AND JAPAN.
RULING:
The Court denies the petition for lack of merit.
There is no question that a license to practice medicine is a privilege or franchise granted by the government.
It is a right that is earned through years of education and training, and which requires that one must first
secure a license from the state through professional board examinations.
[T]he regulation of the practice of medicine in all its branches has long been recognized as a reasonable
method of protecting the health and safety of the public. That the power to regulate and control the
practice of medicine includes the power to regulate admission to the ranks of those authorized to practice
medicine, is also well recognized. Thus, legislation and administrative regulations requiring those who
wish to practice medicine first to take and pass medical board examinations have long ago been
recognized as valid exercises of governmental power.
As required by the said laws, respondent submitted a copy of the Medical Practitioners Law of Japan, duly
authenticated by the Consul General of the Embassy of the Philippines in Japan, which provides in Articles 2
and 11, thus:
Article 2. Anyone who wants to be medical practitioner must pass the national examination for medical
practitioner and get license from the Minister of Health and Welfare.
xxx
Article 11. No one can take the National Medical Examination except persons who conform to one of the
following items:
1. Persons who finished regular medical courses at a university based on the School Education Laws
(December 26, 1947) and graduated from said university.
2. Persons who passed the preparatory test for the National Medical Examination and practiced clinics

44
and public sanitation more than one year after passing the said test.
3. Persons who graduated from a foreign medical school or acquired medical practitioner license in a
foreign country, and also are recognized to have the same or more academic ability and techniques as
persons stated in item 1 and item 2 of this article.

R.A. No. 2382, which provides who may be candidates for the medical board examinations, merely requires
a foreign citizen to submit competent and conclusive documentary evidence, confirmed by the Department of
Foreign Affairs (DFA), showing that his country’s existing laws permit citizens of the Philippines to practice
medicine under the same rules and regulations governing citizens thereof.
PRC is authorized to prescribe additional requirements or grant certain privileges to foreigners seeking
registration in the Philippines if the same privileges are granted to or some additional requirements are
required of citizens of the Philippines in acquiring the same certificates in his country.
 Nowhere in said statutes is it stated that the foreign applicant must show that the conditions for
the practice of medicine in said country are practical and attainable by Filipinos. Neither is it
stated that it must first be proven that a Filipino has been granted license and allowed to practice
his profession in said country before a foreign applicant may be given license to practice in the
Philippines.
 It is enough that the laws in the foreign country permit a Filipino to get license and practice
therein. Requiring respondent to prove first that a Filipino has already been granted license and is
actually practicing therein unduly expands the requirements provided for under R.A. No. 2382 and P.D.
No. 223.
 Indeed, to be granted the privilege to practice medicine, the applicant must show that he possesses all the
qualifications and none of the disqualifications. It must also appear that he has fully complied with all
the conditions and requirements imposed by the law and the licensing authority.
 In this case, there is no doubt as to the competence and qualifications of respondent. He finished his
medical degree from Bicol Christian College of Medicine. He completed a one-year post graduate
internship training at the Jose Reyes Memorial Medical Center, a government hospital. Then he passed
the Medical Board Examinations which was given on August 8, 1992 with a general average of 81.83,
with scores higher than 80 in 9 of the 12 subjects.

45
15. MATAAS NA LUPA TENANTS ASSOCIATION v. CARLOS DIMAYUGA[ GR No. L-32049,
Jun 25, 1984
FACTS: For more than ten years prior to 1959, the petitioners have been occupants of a parcel of
land (with their 110 houses built thereon-110 tenant families) formerly owned by Vda. de Gabriel to whom
petitioners have been paying their rents for the lease thereof, but who, on May 14, 1968, without notice to
petitioners, sold the same to respondent Dimayuga, who in turn mortgaged the same to her for the balance of
the purchase price. On the discovery of the sale the petitioner filed a complaint for the exercise of their
preferential rights before the CFI. And that pursuant to R.A. 1162, as amended by R.A. 2342 a parcel of
land in Manila and suburbs , with at least 50 houses of tenants erected thereon and actually leased to said
tenants for at least 10 years prior to June 20 , 1959, may not be sold by the land owner to any person
other than such tenants, unless the latter renounced their rights in a public instrument. Which means,
respondent Vda. de Gabriel sold the land to respondent Dimayuga without the said tenants-appellants having
renounced their preferential rights in a public instrument.
Their complaint also states that since the aforesaid contract of sale is expressly prohibited by law, the
same be declared null and void and for Vda. De Gabriel to execute a deed of sale in their favor because they
are likewise willing to purchase said land at the same price and on the same terms and conditions observed in
the contract of sale with respondent Dimayuga.
On January 31, 1969, respondent Vda. De Gabriel filed a motion to dismiss on the ground that the
complaint is not a land estate and not being such, the same cannot be expropriated and that no preferential
rights can be availed of by the tenants. On February 6, 1969, Dimayuga filed his answer admitting therein
certain factual allegations, denied some averments, interposed the affirmative defense that plaintiffs had no
personality to initiate the action, that the subject complaint stated no cause of action against respondent and
prayed for the dismissal of the complaint and other remedies. Plaintiffs filed their opposition to the motion to
dismiss, maintaining that R.A. 1162, as amended by R.A. 2342 does not refer to landed estates, but to any
piece of land occupied by more than 50 families leasing the same for more than 10 years prior to June 20,
1959; that their preferential right is independent of the expropriability of the land; that therefore, said
rights may be exercised even if land is not expropriable pursuant to the police power of the State for the
general welfare.
On October 30, 1969, the CFI issued the subject order which found respondent's motion to dismiss
well-taken and thereby dismisses complaint. After a series of motions, reply, rejoinder, surrejoinder, and
answer between both parties, the lower court issued it's order of May 11, 1970 dismissing petitioners appeal.
Petitioner thus resorted to this petition.
ISSUES: Whether or not the contract of sale is null and void.
Whether or not the petitioners may invoke their preferential rights as tenants.

HELD: The Court finds that the said sale was made illegally and therefore void.
The court also finds that petitioners' case falls within the law thus they may invoke their preferential
right.
The R.A. 1162 as amended by R.A. 2342 and 3516 set forth the following conditions-that of offering
first the sale of the land to petitioners and the latter's renunciation in a public instrument-were not met when
the land was sold to respondent Dimayuga. Evidently, said sale is illegal and therefore void.

The 1973 Constitution section 6, article II emphasizes the stewardship


concept that such private property is supposed to be held by the
individual only as trustee for the people in general, who are its real
owners.

As a mere steward, the individual must exercise his right to the property not for his own exclusive and selfish
benefit but for the good of the entire community. P.D. 1157 “Proclaiming Urban Land Reform in the
Philippines and Providing for the Implenting Machinery thereof.” superseded R.A. 1152, 2342,3516. This
decree is firmly based on sec. 6 of art. II of the 1973 constitution undoubtedly adopts and crystallizes the
greater number of people criterion when it speaks of tenants and residents in declared urban land reform
zones or areas without mention of the land area covered by such zones.
The focus therefore, is on people who would benefit and not on the size of the land involved. Under section
6 of which also states that tenant-families have been vested the right of first refusal to purchase of the land
within a reasonable time and reasonable price subject to the rules and regulations of the Ministry of Human
Settlements. It is further supported by PD 1967 which evidently include Mataas na Lupa, the land in
controversy within the Urban Land Reform Zone.
The order issued by the CFI is hereby set aside and the Ministry of Human Settlements is hereby
directed to facilitate and administer the implementation of the rights of the petitioner. Cost against
respondents.

46
16. Agan v PIATCO G.R. No. 155001 May 5, 2003

Puno
Nature of the case: Special Civil Action in the Supreme Court. Prohibition.

Facts:

In August 1989 the DOTC had a study conducted to determine whether or not the present Ninoy Aquino International
Airport (NAIA) can cope with traffic development up to the year 2010. A draft final report was submitted to the DOTC in
December 1989.

Four years later, in 1993, six Filipino-Chinese business leaders met with then President Fidel Ramos to explore the
possibility of investing in the construction and operation of a new international airport terminal. The six later formed the
Asia’s Emerging Dragon Corp (AEDC) which submitted an unsolicited proposal for the development of NAIA
International Passenger Terminal III more than a year after the first meeting with Ramos, in October 1994.
In March 1995, the DOTC endorsed the AEDC proposal to the National Economic and Development Authority (NEDA).
In January 1996, NEDA passed Board Resolution No. 2 which approved the NAIA IPT III project.
In June 1996, the DOTC published an invitation for competitive bidding in two daily newspapers, as required by law (sec
4-A of RA6957). The alternative bidders had to submit three envelopes. The first contains the prequalification
documents, the second the technical proposal, and the third the financial proposal of the proponent.

The bidding was scheduled on September 20, 1996.

The bid documents issued by the Prequalification Bids and Awards Committee said the proponent must have adequate
capability to sustain the financing requirement for the engineering, design, construction, operation, and maintenance
phases of the project. The proponent must have an equity that is at least 30% of the project cost, and be able to secure
external financing for the project.

Government was also guaranteed a five percent share in the gross revenue of the project for the first five years; 7.5% in
the next 10 years, and 10% in the next 10 years. This would be in addition to a fixed annual guaranteed payment to the
government.
The basis for the prequalification shall be the proponent’s compliance with the minimum technical and financial
requirements provided in the bid documents and the IRR of the BOT (build operate and transfer) Law.
The bid documents allowed amendments to the draft concession agreement, but said that these should cover only items
that would not materially affect the preparation of the proponent’s proposal.

In August 1996, during the second pre-bid conference, the PBAC made several clarifications, upon the request of People’s
Air Cargo & Warehousing Co. Inc (PAIRCARGO), which wanted to challenge the AEDC bid.
The PBAC said the list of revenue sources mentioned in the bid documents were “merely indicative.” The project
proponent may add other revenue sources, subject to approval by the DOTC/MIAA. Also, only fees and charges
denominated as “public utility fees” would be subject to regulation, and these could still be revised, because the PBAC
has a pending query with the justice department.

In September 1996, PBAC issued a bid bulletin in which it said that since PAIRCARGO could not meet the required
minimum equity prescribed in the bid documents, it would accept instead an audited financial statement of the financial
capability of all member companies of the consortium.
47
In September 1996, PAIRCARGO submitted their competitive proposal to the PBAC. The first envelope, containing the
prequalification documents, was opened on September 23, 1996, and PBAC prequalified the PAIRCARGO consortium
the following day.

On September 26, AEDC filed with PBAC its reservations regarding PAIRCARGo, noting the “lack of corporate
approvals and financial capability of PAIRCARGO.” For one, PAIRCARGO included in the computation of its financial
capability the total net worth of Security Bank, when the Banking Law limits to 15% the total investment that a bank may
make on one project. It also questioned the appointment of Lufthansa as facility operator, because Philippine laws limit to
Filipinos the operation of a public utility.

The PBAC, however, said on October 2, 1996 that based on the documents submitted and the prequalification criteria,
PAIRCARGO was prequalified. The DOTC secretary approved PBAC’s findings.

The AEDC reiterated its objections two more times.

On October 16, the third envelope containing the financial proposals were opened, and PAIRCARGO had offered to pay
the government higher.

Both PAIRCARGO and AEDC offered to build the NAIA Passenger Terminal III for at least $350 million at no cost to
the government and to pay the government a 5% share in gross revenues for the first five years of operation; a 7.5% share
in gross revenues for the next 10 years of operation; and a 10% share in gross revenues for the last 10 years of operation.
In addition to this, AEDC offered to pay the government P135 million as guaranteed payment for 27 years. Paircargo
Consortium offered a total of P17.75 billion for the same period.

PBAC informed AEDC it had accepted Paircago’s price proposal, and given AEDC 30 working days to match the bid.
When AEDC failed to do so, the DOTC issued a notice on December 11 1996 regarding AEDC’s failure to match the
proposal.

In February 1997, Paircargo Consortium incorporated into Philippine International Airport Terminals Co Inc
(PIATCO).
AEDC protested the alleged undue preference given to PIATCO and reiterated its objections regarding the
prequalification of PIATCO. In April 1997, it filed before the Pasig RTC a petition for declaration of nullity of the
proceedings, mandamus and injunction against the DOTC secretary, the PBAC chair and its voting members, and the
chair of the PBAC technical committee.

On April 17, the NEDA ICC conducted an ad referendum to facilitate the approval of the BOT agreement between the
DOTC and PIATCO. Because there were only four instead of the required six signatures, the NEDA merely noted the
agreement.

On July 9, 1997, the DOTC issued the notice of award for the project to PIATCO.
The Concession Agreement was signed on July 12, 1997, granting PIATCO the franchise to operate and maintain
the NAIA Passenger Terminal III for 25 years, with an option to renew for a period not exceeding 25 years.
PIATCO was allowed to collect fees, rentals and other charges in accordance with the rates or schedules in the 1997
Concession Agreement. At the end of the concession period, PIATCO will transfer the airport to MIAA.
In November 1998, the government and PIATCO signed an Amended and Restated Concession Agreement
(ARCA). The ARCA amended provisions on the special obligations of the government, the exclusivity of the franchise
48
given to PIATCO, the proceeds of the insurance, the taxes, duties and other charges that may be levied PIATCO, and the
provisions on the termination of the contract. Three more supplements to the ARCA were signed afterwards: in August
1999, in September 2000, and in June 2001.

The first redefined revenues, required government to construct an access road connecting NAIA II and III, and
added to the special obligations of government.

The second supplement required government to clear structures at the construction site and to pay PIATCO for
these. The third provided for PIATCO’s obligations regarding the construction of the surface road connecting Terminals II
and III.

In September 2002, workers of the international airline service providers filed before the Supreme Court a petition for
prohibition enjoining the enforcement of the agreements. They said the transfer to NAIA III could cost them their jobs,
since under the agreements, PIATCO is not required to honor MIAA’s existing concession contracts with various
service providers for international airline airport services.

In October 2002, the service providers filed a motion for intervention and a petition in intervention, joining the cause of
the petitioning workers.

Three congressmen – Salacnib Baterina, Clavel Martinez and Constantino Jaraula – filed a similar petition shortly after.
In November 2002, several MIAA employees also filed a petition questioning the legality of the agreements.
In December 2002, another group of congressmen – Jacinto Paras, Rafael Nantes, Eduardo Zialcita, Willie Villarama,
Prospero Nograles, Prospero Pichay Jr., Harlin Cast Abayon and Benasing Macarambon – filed their comment in
intervention defending the validity of the agreements and praying for the dismissal of the petitions.
On December 10, 2002, the court heard the case on oral argument, the required the parties to file their respective
memoranda, and to explore the possibility of arbitration as provided in the challenged contracts.

In their consolidated memorandum, the Office of the Solicitor General and the Office of the Government Corporate
Counsel prayed that the petitions be given due course and that the 1997 Concession Agreement, the ARCA and the
supplements be declared void for being contrary to the Constitution, the BOT Law and its implementing rules and
regulations.
In March 2003, PIATCO commenced arbitration proceedings before the International Chamber of Commerce,
International Court of Arbitration.

Issue:

Did the PIATCO agreements – the 1997 Concession Agreement, the ARCA, and the three supplemental agreements –
violate the Constitution and the BOT Law?

Held/Decision: YES.

Ratio:
In the first place, PIATCO was not a qualified bidder. The minimum project cost was estimated to be P9.183 billion,
which meant that Paircargo Consortium had to prove it could provide at least P2.755 billion. Paircargo’s audited financial
statement for 1994 showed it had a net worth of P3.123 billion, but that was because it included in the computation the
total net worth of Security Bank, which as of 1995 was at P3.523 billion. Since banks are allowed to invest only 15% of it
entire net worth, Security Bank could only invest P528 million, which brings down Paircargo’s equity to P558.384
million, or only 6% of the project cost.

49
“Disregarding the investment ceilings provided by applicable law would not result in a proper evaluation of whether or
not a bidder is pre-qualified to undertake the project as for all intents and purposes, such ceiling or legal restriction
determines the true maximum amount which a bidder may invest in the project…If the maximum amount of equity that a
bidder may invest in the project at the time the bids are submitted falls short of the minimum amounts required to be put
up by the bidder, said bidder should be properly disqualified…we hold that Paircargo Consortium was not a qualified
bidder. Thus the award of the contract by the PBAC to the Paircargo consortium is null and void.”

Other issues:
1. The signed agreement was different from the draft that was bidded on. Under the law, substantial changes require
another bidding.

The three principles in public bidding are:


1. an offer to the public;
2. opportunity for competition; and
3. basis for the exact comparison of bids.

Changing the parameters would change the agreement, which might have changed the technical and financial parameters
of other bidders had they known that such terms were available.

The 1997 Concession Agreement signed between PIATCO and the government was substantially different from the
draft concession agreement that was bidded on. While it was a draft and was expected to amended from time to time,
the PBAC bid bulletin also said that the amendments should only cover items that would not materially affect the
preparation of the proposal. The changes should not be substantial or material enough to alter the basic parameters of
the contract, and constitute a denial to the other bidders of the opportunity to bid on the same terms.

There were two main differences between the draft agreement and the one that was signed. These concerned the fees that
may be imposed and collected by PIATCO, and the extent of control and regulation that MIAA has over the fees
that PIATCO will charge.

The draft agreement classified aircraft parking and tacking fees, groundhandling fees, rentals and airlines offices, check-in
counter rentals and porterage fees under those that are regulated – subject to periodic adjustment of once every two years
and in accordance to a certain formula. The signed agreement said fees subject to MIAA approval are “public utility fees”
and took out groundhandling and rentals and airlines offices from the list. There was also “an obvious relaxation of the
extent of control and regulation by MIAA with respect to the particular fees that may be charged by PIATCO.” The
signed agreement also allowed PIATCO to charge in US dollars, while paying the government in pesos.

“When taken as a whole, the changes under the 1997 Concession Agreement with respect to reduction in the types of fees
that are subject to MIAA regulation and the relaxation of such regulation with respect to other fees are significant
amendments that substantially distinguish the draft Concession Agreement from the 1997 Concession Agreement. The
1997 Concession Agreement, in this respect, clearly gives PIATCO more favorable terms than what was available to other
bidders at the time the contract was bidded out. It is not very difficult to see that the changes in the 1997 Concession
Agreement translate to direct and concrete financial advantages for PIATCO which were not available at the time the
contract was offered for bidding. It cannot be denied that under the 1997 Concession Agreement only “Public Utility
Revenues” are subject to MIAA regulation. Adjustments of all other fees imposed and collected by PIATCO are entirely
within its control. Moreover, with respect to terminal fees, under the 1997 Concession Agreement, the same is further

50
subject to “Interim Adjustments” not previously stipulated in the draft Concession Agreement. Finally, the change in the
currency stipulated for “Public Utility Revenues” under the 1997 Concession Agreement, except terminal fees, gives
PIATCO an added benefit which was not available at the time of bidding.

2. Under the draft Concession Agreement, default by PIATCO of its obligations does not result in the assumption by
government of these liabilities. Under the signed agreement, default by PIATCO of its loans used to finance the project
eventually leads to government assumption of the liability for the loans.

This is in violation of the BOT Law, which prohibits direct government guarantees. “If a proposal can be denied by
reason of the existence of direct government guarantee, then its inclusion in the contract executed after the said proposal
has been accepted is likewise sufficient to invalidate the contract itself…To declare the PIATCO contracts valid despite
the clear statutory prohibition against a direct government guarantee would not only make a mockery of what the BOT
Law seeks to prevent – which is to expose the government to the risk of incurring a monetary obligation resulting
from a contract of loan between the project proponent and its lenders and to which the Government is not a party to –
but would also render the BOT Law useless for what it seeks to achieve – to make use of the resources of the private
sector in the financing, operation and maintenance of infrastructure and development projects which are necessary for
national growth and development but which the government, unfortunately, could ill-afford to finance at this point in
time.”

3. Sec. 5.10 of the 1997 Concession Agreement violates Article XII, Sec. 12 of the 1987 Constitution.

The Constitutional provision allows for temporary takeover of public facilities in times of national emergency.
Since the takeover is temporary and extends only to the operation of the business and not the ownership, government is
not required to compensate the owner. Neither can the owner claim just compensation for the use of the business and its
properties because the takeover is in exercise of the State’s police power and not of its power of eminent domain.
The 1997 Concession Agreement, on the other hand, says that in the event of a takeover, “Concessionaire shall be
entitled to reasonable compensation for the duration of the temporary takeover…”

“PIATCO cannot, by mere contractual stipulation, contravene the Constitutional provision on temporary government
and obligate the government pay reasonable cost for the use of Terminal and/or Terminal Complex.
Police power is the most essential, insistent, and illimitable of powers. Its exercise must not be unreasonably hampered
nor its exercise be a source of obligation by the government in the absence of damage due to arbitrariness of its exercise.

4. The 1987 Constitution strictly regulates monopolies. Art XII, Sec. 19 says: The State shall regulate or prohibit
monopolies when the public interest so requires.

The 1997 Concession Agreement gave PIATCO the exclusive right to operate a commercial international passenger
terminal within the island of Luzon, with the exception of already existing terminals such as those in the Subic Bay
Freeport, Clark Special Economic Zone, and in Laoag City. This privilege, however, is subject to reasonable regulation
and supervision and should not violate the rights of third parties.

There are service providers at the NAIA I with existing contracts with the MIAA valid until 2010; since the 1997
Concession Agreement says PIATCO is not bound to honor existing contracts with MIAA, transferring operations from

51
NAIA I to NAIA III would unduly prejudice them. “PIATCO cannot, by law and certainly not by contract, render a
valid and binding contract nugatory. PIATCO, by the mere expedient claiming an exclusive right to operate,
cannot require the Government to break its contractual obligations to the service providers.”

IN SUM, THIS COURT RULES THAT IN VIEW OF THE ABSENCE OF THE REQUISITE FINANCIAL CAPACITY
OF THE PAIRCARGO CONSORTIUM, PREDECESSOR OF RESPONDENT PIATCO, THE AWARD BY THE
PBAC OF THE CONTRACT FOR THE CONSTRUCTION, OPERATION AND MAINTENANCE OF THE NAIA IPT
III IS NULL AND VOID. FURTHER, CONSIDERING THAT THE 1997 CONCESSION AGREEMENT
CONTAINS MATERIAL AND SUBSTANTIAL AMENDMENTS, WHICH AMENDMENTS HAD THE EFFECT
OF CONVERTING THE 1997 CONCESSION AGREEMENT INTO AN ENTIRELY DIFFERENT AGREEMENT
FROM THE CONTRACT BIDDED UPON, THE 1997 CONCESSION AGREEMENT IS SIMILARLY NULL AND
VOID FOR BEING CONTRARY TO PUBLIC POLICY. THE PROVISIONS UNDER SECTIONS 4.04(B) AND (C)
IN RELATION TO SECTION 1.06 OF THE 1997 CONCESSION AGREEMENT AND SECTION 4.04 (C) IN
RELATION TO SECTION 1.06 OF THE ARCA, WHICH CONSTITUTE A DIRECT GOVERNMENT
GUARANTEE EXPRESSLY PROHIBITED BY, AMONG OTHERS, THE BOT LAW AND ITS
IMPLEMENTING RULES AND REGULATIONS ARE ALSO NULL AND VOID. THE SUPPLEMENTS, BEING
ACCESSORY CONTRACTS TO THE ARCA, ARE LIKEWISE NULL AND VOID.

JUDGMENT: The 1997 Concession Agreement, the Amended and Restated Concession Agreement and the Supplements
thereto are set aside for being null and void.

***VITUG: separate opinion – court has no jurisdiction. Petition prays for nullification of contract and does not involve
judicial, quasi-judicial or ministerial functions. The parties allege contentious evidentiary facts and “it would be difficult
to decide the jurisdictional issue on the basis of the contradictory factual submissions made by the parties.”

***issues on standing: piatco tried to raise the issue of jurisdiction saying they had filed a case with the international
chamber of commerce, international court of arbitration for arbitration. The court however pointed out that it includes
multiple contracts and not all of the petitioners are parties to the contract.

52

You might also like